Sie sind auf Seite 1von 31

Chapter 10Standard Costing: A Managerial Control Tool

Standard costs are developed for direct materials, direct


labor, and variable overhead only.

TRUE/FALSE
1.

2.

Managers develop quantity standards when they decide ANS:


F
what amount of input should be used per unit of output. No. Standard costs are assigned for direct materials,
direct labor, and variable and fixed overhead.
ANS:
T
Managers develop price standards when they determine The standard quantity of materials allowed can be
what amount should be paid for the quantity of input to calculated by multiplying the unit labor standard by the
be used.
actual output.
ANS:

3.

The standard cost per unit of output for a particular


input is calculated by multiplying the standard input
price by the standard input allowed per unit of output
produced.
ANS:

4.

5.

6.

7.

8.

10.

To compute the standard direct labor hours allowed,


multiply the unit labor standard by the actual output.

ANS:
T
Engineering studies are often too rigorous and may not
be achievable by operating personnel.
The standard cost sheet provides the input standards
needed to compute the total amount of inputs allowed
ANS:
T
for the actual output, an essential component in
computing efficiency variances.
Ideal standards can be achieved under efficient
operating conditions.
ANS:
T
ANS:
F
No. Currently attainable standards can be achieved
under efficient operating conditions. Ideal standards
demand maximum efficiency and can be achieved only
if everything operates perfectly.

The standard unit cost is developed before the standard


costs for direct materials, direct labor, and overhead
can be set.

Ideal standards can be achieved only if everything


operates perfectly, meaning that they do not allow for
any machine breakdowns, slack, etc.

The unit standard quantity of inputs is vital to the


computation of total amount of inputs allowed for the
actual output and efficiency variances.

ANS:

ANS:

ANS:

Currently attainable standards offer the most behavioral The total budget variance is the difference between the
benefits because higher performance levels are
actual cost of the input and its planned cost.
attained through challenging, yet achievable, standards.
T

ANS:

Currently attainable standards can be achieved under


efficient operating conditions.

The actual quantity of input at the standard price less


than the standard quantity of input at the standard
price equals the usage variance.

ANS:

ANS:

One reason for adopting a standard cost system is to


make product costing easier.
ANS:

11.

ANS:
F
The standard quantity of materials allowed can be
calculated by multiplying the unit quantity standard by
the actual output.

ANS:
T
In setting standards, historical experience should be
used with caution because it can perpetuate operating The quantity of each input that should be used to
inefficiencies.
produce one unit of output is documented on the
standard cost sheet.
ANS:
T

ANS:
9.

The benefits of operational control under a standard


cost system can extend to all manufacturing
environments.

The actual quantity of input at the actual price less the


actual quantity of input at the standard price is the
price variance.
ANS:

An unfavorable usage variance would occur when the


actual usage of inputs is greater than the standard
usage.

ANS:
F
No. Manufacturing environments that focus on
ANS:
T
continuous improvement and JIT purchasing and
manufacturing do not realize the benefits of operational An unfavorable price variance occurs whenever the
control in a standard cost system.
actual prices are greater than the standard prices.

ANS:
24.

An acceptable range is established in order to


determine if whether variances are significant. The
acceptable range is the standard, plus or minus an
allowable deviation.
ANS:

25.

d.
e.
f.
g.

Standard Cost Sheet


Upper Control Limit
Currently Attainable Standards
Kaizen Standards

Standards of perfection that require absolute efficiency.


Standards that are rigorous but achievable and reflect
reasonable efficiency.

The sum of the price and usage variances will add up to


These reflect the amount that should be paid for the
the total materials variance only if the materials
quantity of input to be used.
purchased is equal to the materials used.
ANS:

These reflect the amount of input that should be used


per unit of output.

26.

The materials price variance is computed using the


A tool used to provide the production data needed to
actual quantity of materials used, and the materials
usage variance is computed using the actual quantity of calculate the standard unit cost.
materials purchased.
This is the standard plus the allowable deviation when
determining whether variances are significant.
ANS:
F
No. The materials price variance is computed using the
This reflects the planned improvement that is set, which
actual quantity of materials purchased, and the
materials usage variance is computed using the actual will help reduce nonvalue-added costs.
quantity of materials used.
ANS:
B

27.

ANS:
For better control, the materials price variance is
computed using actual quantity of materials purchased.
ANS:
ANS:
T
ANS:
Since it is better to have information on variances
earlier rather than later, the materials price variance
uses the actual quantity of materials purchased rather ANS:
than the actual quantity of materials used.
ANS:

28.

29.

30.

F
C
A
D
E

G
The sum of the labor rate and labor efficiency variances ANS:
will always add up to the total labor variance.
Match the variance with its correct calculation.
a. Actual Quantity Actual Price
ANS:
T
b. (Actual Hours Actual Rate) (Standard Hours
Standard Rate)
Kaizen costing provides fixed standards which reflect
c. (Actual Quantity Actual Price) (Standard
continuous improvement efforts.
Quantity Standard Price)
d. (Actual Hours Standard Hours) Standard
ANS:
F
Rate
Thus, kaizen costing differs from traditional standard
e. (Actual Price Standard Price) Actual
costing in that the standard changes frequently,
Quantity
reflecting continuous improvement efforts.
f. Standard Quantity Standard Price
A kaizen standard reflects the realized improvements g. (Actual Rate Standard Rate) Actual Hours
h. (Actual Quantity Standard Quantity)
for the past periods and a search for more
Standard Price
improvements for the future.
ANS:
T
Setting this new level as a minimum standard for future Actual Costs
performance locks in the realized improvements and
Budgeted Costs
initiates simultaneously the maintenance cycle and a
search for additional improvement opportunities
Total Materials Variance

31.

Favorable variances are credits and unfavorable


variances are debits.
ANS:

MATCHING
Match each item with the correct statement below.
a. Quantity Standards
b. Ideal Standards
c. Price Standards

Materials Price Variance


Materials Usage Variance
Labor Rate Variance
Labor Efficiency Variance
Total Direct Labor Variance
ANS:

9.

ANS:

The ______________________ can be used to compute the


total amount of inputs allowed for the actual output.

10.

ANS:

ANS:

11.

ANS:

12.

ANS:

___________________ is calculated by multiplying the unit


labor standard by the actual output.

13.

ANS:

ANS:

14.

ANS:

15.

ANS:

COMPLETION
1.

ANS:
2.

Cost control

The amount of input that should be used per unit of


output is known as the _______________.
ANS:
quantity decision
standard quantity

3.

The amount that should be paid for the quantity of the


input to be used is known as the ______________.
ANS:
pricing decision
standard price
___________________ can provide an initial guideline for
setting standards, but should be used with caution
because they can perpetuate existing inefficiencies.
ANS:

Historical experiences

ANS:
Price variance
rate variance
_________________ occur whenever actual prices or actual
usage of inputs are greater than standard prices or
standard usage.
ANS:

6.

7.

total variance for materials

The ____________________ measures the difference


between what should have been paid for raw materials
and what was actually paid.
materials price variance

Standards are set by using historical experiences,


___________________, and input from operating personnel, The _____________________ measures the difference
marketing, and accounting.
between the direct materials actually used and the
direct materials that should have been used for the
ANS:
engineering studies
actual output.
ANS:
materials usage variance
________________ demands maximum efficiency and can
be achieved only if everything operates perfectly.
The _______________ computes the difference between
what was paid to direct laborers and what should have
ANS:
Ideal standards
been paid.
ANS:
labor rate variance
In a ____________________, costs are assigned to products
using quantity and price standards for all three
The ___________________ measures the difference
manufacturing costs: direct materials, direct labor, and between the labor hours that were actually used and
overhead.
the labor hours that should have been used.
ANS:

8.

Unfavorable variances

The ____________________ measures the difference


between the actual costs of materials and their
budgeted costs for actual level of activity.

ANS:
5.

total budget variance

____________________ is the difference between the


actual and standard unit price of an input multiplied by
the number of inputs used.

ANS:
4.

Standard hours allowed

The ____________________ is the difference between the


actual cost of the input and its planned cost.
ANS:

_______________ often means the difference between


success and failure or between above-average profits
and lesser profits.

unit quantity standards

standard costing system

The __________________ provides the products data


needed to calculate the standard unit cost.
ANS:

standard cost sheet

ANS:

labor efficiency variance

______________ focuses on the continuous reduction of


the manufacturing costs of existing products and
processes.
ANS:

Kaizen costing

b. They are often achievable by operating


A ______________ is the difference between the sales
personnel.
price needed to capture a predetermined market share c. They provide very rigorous guidelines.
and the desired per-unit profit.
d. All of these statements are true.
e. More than two of these statements are true.
ANS:
target cost
ANS:
B
MULTIPLE CHOICE
Engineering studies are typically so rigorous they are
often not achievable by operating personnel.
1. Standards based on the amount of input that should be
used per unit of output are called
In setting price standards for materials and labor,
a. quantity standards.
a. the purchasing department must consider
b. price standards.
discounts, freight, and quality.
c. ideal standards.
b. personnel must consider payroll taxes, fringe
d. currently attainable standards.
benefits, and qualifications.
e. kaizen standards.
c. it is the joint responsibility of operations,
purchasing, personnel, and accounting.
d. All of these.
e. None of these.
ANS:
A
20.

2.

Price standards are based on


a. the amount of input that should be used per unit
of output.
b. the amount that should be paid for the total
quantity of input to be used.
c. the amount that should be paid per unit of
output.
d. the amount that should be paid per unit of input
purchased.
e. None of these.

Ideal standards
a. do not allow for machine breakdowns, slack, or
lack of skill (even momentarily).
b. demand maximum efficiency.
c. can be achieved only if everything operates
perfectly.
d. All of these.
e. None of these.

Which of the following is true regarding currently


attainable standards?
a. They can be achieved under efficient operating
conditions.
ANS:
B
Price standards are based on the pricing decision, whichb. Allowance is made for normal breakdowns,
interruptions, etc.
is the amount that should be paid for the quantity of
c. They are challenging but achievable.
input to be used.
d. They tend to achieve higher performance levels
from personnel.
e. All of these.
3. The sources of quantitative standards include
a. historical experience.
ANS:
E
b. engineering studies.
c. input from operating personnel.
Standard cost systems are adopted
d. historical experience, engineering studies, and
a. to improve planning and control.
input from operating personnel.
b. to facilitate product costing.
e. None of these.
c. to improve planning and control, and to facilitate
product costing.
d. to enhance the operational control of firms that
ANS:
D
emphasize continuous improvement.
e. for all of these reasons.
4. Which of the following is true regarding historical
experience in standard setting?
a. It provides very rigorous guidelines.
ANS:
C
b. Operating personnel may not be able to achieve
The benefits of operational control in a standard cost
operating standards based on historical
system may not extend to the manufacturing
experience.
environments that emphasize continuous improvement
c. It should be used with caution because it can
and just-in-time purchasing and manufacturing.
perpetuate inefficiencies.
d. Standards based on historical experience are
Standard cost systems can enhance operational control
better than standards based on engineering
through the use of
studies.
a. efficiency variances which indicate the need for
e. None of these.
corrective action.
b. price variances which indicate the need for
better spending control.
ANS:
C
c. standard costs which indicate the desired cost of
Historical experience should be used with caution
a unit of input.
because processes are often operating inefficiently.
d. actual costs which indicate the price received for
Using standards based on past experience can
units sold.
perpetuate the inefficiencies.
e. All of these.
5. Which of the following is not true regarding engineering
ANS:
A
studies?
a. They can determine the most efficient way to
operate.

11.

Which of the following is true regarding standard cost


and direct labor to products using a
systems in manufacturing environments that emphasize
predetermined rate.
continuous improvement and just-in-time
c. In a normal costing system overhead is assigned
manufacturing and purchasing?
using a budgeted rate and actual activity.
a. The standard cost system enhances the
d. A normal costing system has less capacity for
operational control.
control than a standard costing system.
b. The materials price variance may encourage the
e. All of these statements are true.
purchasing department to buy in smaller
quantities to reduce inventories.
c. Variances can be computed and presented in
ANS:
B
reports to higher-level managers.
A normal costing system assigns direct materials and
d. The operational level will benefit from the
direct labor to products using actual costs.
detailed computation of variances.
e. None of these.
PTS:
1
DIF:
Difficulty: Moderate
OBJ:
LO: 10-1
NAT:
BUSPROG: Analytic
ANS:
C
STA:
AICPA: FN-Measurement | IMA: Performance
Although the benefits of operational control may not
Measurement | ACBSP: APC-27-Managerial Accounting
extend to these manufacturing environments, variances Features/Costs
KEY:
can still be computed and presented in reports to
Bloom's: Comprehension
higher-level managers so they can monitor the financial NOT:
2 min.
dimension.
Which of the following is not an advantage of standard
PTS:
1
DIF:
Difficulty: Moderate
costing over normal costing and actual costing?
OBJ:
LO: 10-1
a. A greater capacity for control.
NAT:
BUSPROG: Analytic
b. Ability to easily distinguish the FIFO and
STA:
AICPA: FN-Decision Modeling | IMA: Performance
weighted average methods of accounting for
Measurement | ACBSP: APC-27-Managerial Accounting
beginning inventory costs.
Features/Costs
KEY:
c. Computing a unit cost for each equivalent unit
Bloom's: Comprehension
cost category is not necessary.
NOT:
2 min.
d. Providing for readily available unit cost
information.
12. In a standard cost system, costs are assigned to all of e. All of these are advantages of standard costing.
the following, except for
a. direct materials.
b. direct labor.
ANS:
B
c. variable overhead.
There is no need to distinguish between the FIFO and
d. fixed overhead.
weighted average methods of accounting for beginning
e. none of these.
inventory costs.
ANS:
E
PTS:
1
DIF:
Difficulty: Easy
OBJ:
LO: 10-1
NAT:
BUSPROG: Analytic
STA:
AICPA: FN-Measurement | IMA: Performance
Measurement | ACBSP: APC-27-Managerial Accounting
Features/Costs
KEY:
Bloom's: Comprehension
NOT:
2 min.
13.

14.

PTS:
1
DIF:
Difficulty: Moderate
OBJ:
LO: 10-1
NAT:
BUSPROG: Analytic
STA:
AICPA: FN-Measurement | IMA: Performance
Measurement | ACBSP: APC-27-Managerial Accounting
Features/Costs
KEY:
Bloom's: Comprehension
NOT:
2 min.

The standard cost system differs from the actual cost


system in the assignment of
a. direct materials.
b. direct labor.
c. overhead.
d. all of the manufacturing inputs.
e. none of the manufacturing inputs.

The production data needed to calculate the standard


unit cost as well as the underlying details for the
standard cost per unit are provided in
a. the standard cost sheet.
b. the standard production budget.
c. the balance sheet.
d. the standard work-in-process account.
e. None of these.

ANS:
D
PTS:
1
DIF:
Difficulty: Easy
OBJ:
LO: 10-1
NAT:
BUSPROG: Analytic
STA:
AICPA: FN-Measurement | IMA: Performance
Measurement | ACBSP: APC-27-Managerial Accounting
Features/Costs
KEY:
Bloom's: Knowledge
NOT:
2 min.

ANS:
A
PTS:
1
DIF:
Difficulty: Easy
OBJ:
LO: 10-2
NAT:
BUSPROG: Analytic
STA:
AICPA: FN-Measurement | IMA: Performance
Measurement | ACBSP: APC-27-Managerial Accounting
Features/Costs
KEY:
Bloom's: Knowledge
NOT:
2 min.

Which of the following is not true regarding normal


costing systems?
a. A normal costing system predetermines
overhead costs.
b. A normal costing system assigns direct materials

Standard hours allowed are computed using the


equation
a. unit labor standard actual output.
b. unit labor standard standard output.
c. unit labor standard actual input.

d. unit labor standard standard input.


e. not shown here.
ANS:
A
PTS:
1
DIF:
Difficulty: Moderate
OBJ:
LO: 10-2
NAT:
BUSPROG: Analytic
STA:
AICPA: FN-Measurement | IMA: Performance
Measurement | ACBSP: APC-27-Managerial Accounting
Features/Costs
KEY:
Bloom's: Comprehension
NOT:
2 min.

18.

PTS:
1
DIF:
Difficulty: Easy
OBJ:
LO: 10-2
NAT:
BUSPROG: Analytic
STA:
AICPA: FN-Measurement | IMA: Performance
Measurement | ACBSP: APC-27-Managerial Accounting
Features/Costs
KEY:
Bloom's: Application
NOT:
3 min.

Refer to Figure 10-1. Compute the standard number of


The standard quantity of materials allowed is computed sheets of aluminum allowed for a volume of 10,000
by the equation
airplanes.
a. unit quantity standard standard output.
a. 15,000 sheets
b. unit quantity standard actual input.
b. 10,000 sheets
c. unit quantity standard standard input.
c. 7,500 sheets
d. unit quantity standard actual output.
d. 11,250 sheets
e. not shown here.
ANS:
D
PTS:
1
DIF:
Difficulty: Moderate
OBJ:
LO: 10-2
NAT:
BUSPROG: Analytic
STA:
AICPA: FN-Measurement | IMA: Performance
Measurement | ACBSP: APC-27-Managerial Accounting
Features/Costs
KEY:
Bloom's: Comprehension
NOT:
2 min.

19.

SH = Unit labor standard Actual output


SH = 6 10,000 = 60,000.

An accountant would refer to a cost sheet to perform


which of the following actions?
a. Calculate standard cost per unit.
b. Calculate efficiency variances.
c. Calculate the total amount of inputs allowed for
the actual output.
d. All of these.
ANS:
D
PTS:
1
DIF:
Difficulty: Easy
OBJ:
LO: 10-2
NAT:
BUSPROG: Analytic
STA:
AICPA: FN-Measurement | IMA: Performance
Measurement | ACBSP: APC-27-Managerial Accounting
Features/Costs
KEY:
Bloom's: Knowledge
NOT:
2 min.

ANS:
B
SQ = Unit quantity standard Actual output.
SQ = 10,000 1 = 10,000 sheets.
PTS:
1
DIF:
Difficulty: Easy
OBJ:
LO: 10-2
NAT:
BUSPROG: Analytic
STA:
AICPA: FN-Measurement | IMA: Performance
Measurement | ACBSP: APC-27-Managerial Accounting
Features/Costs
KEY:
Bloom's: Application
NOT:
3 min.
Variances indicate
a. that actual performance is not going according
to plan.
b. the cause of the variance.
c. who is responsible for the variance.
d. when the variance should be investigated.
e. none of these.
ANS:
A
PTS:
1
DIF:
Difficulty: Moderate
OBJ:
LO: 10-3
NAT:
BUSPROG: Analytic
STA:
AICPA: FN-Measurement | IMA: Performance
Measurement | ACBSP: APC-27-Managerial Accounting
Features/Costs
KEY:
Bloom's: Knowledge
NOT:
2 min.

Figure 10-1.
Flying High Company manufactures model airplanes.
During the month, it manufactured 10,000 airplanes.
Each one used an average of 6.5 direct labor hours and The difference between the actual cost of the input and
an average of 1.5 sheets of aluminum. It normally
its planned cost is
manufactures 7,500 airplanes. Materials and labor
a. the total budget variance.
standards for making the airplanes are:
b. the usage variance.
c. the price variance.
Direct Materials (1 sheet of aluminum @
$10.00
d. the efficiency variance.
$10.00)
e. the budget variance.
Direct Materials (other accessories @
8.75
$8.75)
Direct Labor (6 hours @ $7.00)
42.00
ANS:
A
PTS:
1
DIF:
Difficulty: Easy
OBJ:
LO: 10-3
NAT:
BUSPROG: Analytic
20. Refer to Figure 10-1. Compute the standard hours
STA:
AICPA: FN-Measurement | IMA: Performance
allowed for a volume of 10,000 airplanes.
Measurement | ACBSP: APC-27-Managerial Accounting
a. 60,000 hours
Features/Costs
KEY:
b. 420,000 hours
Bloom's: Knowledge
c. 70,000 hours
NOT:
2 min.
d. 65,000 hours
Which of the following is true concerning the materials
price variance?
ANS:
A
a. It is the difference between the actual and

b.
c.
d.
e.

standard unit price of an input multiplied by the


number of inputs used.
It is the difference between the actual and
standard unit price of an output multiplied by
the number of inputs used.
It is the difference between the actual and
standard unit price of an input multiplied by the
number of inputs purchased.
It is the difference between the actual and
standard unit price of an output multiplied by
the number of inputs purchased.
None of these.

variance analysis on a case-by-case basis.


d. variances are not investigated unless they are
large enough to be of a concern.
e. every variance is investigated.
ANS:
E
Only material variances are investigated.
PTS:
1
DIF:
Difficulty: Moderate
OBJ:
LO: 10-3
NAT:
BUSPROG: Analytic
STA:
AICPA: FN-Measurement | IMA: Performance
Measurement | ACBSP: APC-27-Managerial Accounting
Features/Costs
KEY:
Bloom's: Comprehension
NOT:
2 min.

ANS:
C
PTS:
1
DIF:
Difficulty: Moderate
OBJ:
LO: 10-3
NAT:
BUSPROG: Analytic
STA:
AICPA: FN-Measurement | IMA: Performance
Measurement | ACBSP: APC-27-Managerial Accounting Which of the following is not true concerning control
Features/Costs
KEY:
limits?
Bloom's: Comprehension
a. Control limits are the top and bottom measures
NOT:
2 min.
of the allowable range.
b. The upper control limit is the standard plus the
25. The usage variance is the difference between the actual
allowable deviation.
and standard quantity of inputs
c. The lower control limit is the standard minus the
a. multiplied by the standard unit price of the
allowable deviation.
input.
d. In current practice, control limits are set
b. budgeted multiplied by the standard unit price of
objectively using standard formulas.
the input.
e. Variances that fall outside the control limits are
c. multiplied by the actual unit price of the input.
investigated.
d. purchased multiplied by the actual unit price of
the input.
e. None of these.
ANS:
D
In current practice, control limits are set subjectively,
using past experience, judgment, and intuition.
ANS:
A
PTS:
1
DIF:
Difficulty: Moderate
PTS:
1
DIF:
Difficulty: Moderate
OBJ:
LO: 10-3
NAT:
BUSPROG: Analytic
OBJ:
LO: 10-3
STA:
AICPA: FN-Measurement | IMA: Performance
NAT:
BUSPROG: Analytic
Measurement | ACBSP: APC-27-Managerial Accounting STA:
AICPA: FN-Measurement | IMA: Performance
Features/Costs
KEY:
Measurement | ACBSP: APC-27-Managerial Accounting
Bloom's: Comprehension
Features/Costs
KEY:
NOT:
2 min.
Bloom's: Comprehension
NOT:
2 min.
26. Which of the following is true regarding variances?
a. Unfavorable variances occur whenever actual
Acme Company's standard cost is $500,000. The
prices or actual usage of inputs are greater than
allowable deviation is 10%. Its actual costs for three
standard prices or standard usage.
months are
b. Favorable variances occur whenever actual
prices or actual usage of inputs are greater than
January
$520,000
standard prices or standard usage.
February
$550,000
c. Unfavorable variances are always credits.
March
$575,000
d. Favorable variances are always debits.
e. None of these.
The upper and lower control limits are, respectively,
a. $550,000 and $450,000
b. $500,000 and $450,000
ANS:
A
PTS:
1
DIF:
c. $550,000 and $500,000
Difficulty: Moderate
d. $575,000 and $520,000
OBJ:
LO: 10-3
NAT:
BUSPROG: Analytic
STA:
AICPA: FN-Measurement | IMA: Performance
Measurement | ACBSP: APC-27-Managerial Accounting ANS:
A
Features/Costs
KEY:
The upper control limit is $550,000 ($500,000 +
Bloom's: Comprehension
($500,000 10%)).
NOT:
2 min.
The lower control limit is $450,000 ($500,000
($500,000 10%)).
27. All of the following are true regarding variance
investigation except
PTS:
1
DIF:
Difficulty: Easy
a. the investigation should be undertaken only if
OBJ:
LO: 10-3
the anticipated benefits are greater than the
NAT:
BUSPROG: Analytic
expected costs.
STA:
AICPA: FN-Measurement | IMA: Performance
b. managers must consider whether a variance will
Measurement | ACBSP: APC-27-Managerial Accounting
recur.
Features/Costs
KEY:
c. it is difficult to assess the costs and benefits of
Bloom's: Application

NOT:

3 min.

Which of the following is not true concerning direct


materials variances?
Figure 10-2.
a. The sum of the price and usage variances will
Highland Company's standard cost is $250,000. The
add up to the total materials variance only if the
allowable deviation is 10%. Its actual costs for six
materials purchased is equal to the materials
months are
used.
b. The materials price variance uses the actual
January
$235,000
quantity of materials purchased rather than the
February
220,000
actual quantity of materials used.
March
245,000
c. The materials price variance always uses the
April
265,000
actual quantity of materials used rather than the
May
270,000
actual quantity of materials purchased.
June
280,000
d. The materials usage variance uses the actual
quantity of materials used.
e. Separate materials variances can be computed
30. Refer to Figure 10-2. The upper and lower control limits
for each type of material used.
are, respectively,
a. $250,000 and $225,000
b. $305,000 and $195,000
ANS:
C
c. $275,000 and $250,000
Typically, the quantity of material purchased is used to
d. $275,000 and $225,000
determine the materials price variance.
ANS:
D
The upper control limit is $275,000 ($250,000 +
($250,000 10%)).
The lower control limit is $225,000 ($250,000
($250,000 10%)).

PTS:
1
DIF:
Difficulty: Moderate
OBJ:
LO: 10-4
NAT:
BUSPROG: Analytic
STA:
AICPA: FN-Measurement | IMA: Performance
Measurement | ACBSP: APC-27-Managerial Accounting
Features/Costs
KEY:
Bloom's: Comprehension
NOT:
3 min.

PTS:
1
DIF:
Difficulty: Easy
OBJ:
LO: 10-3
NAT:
BUSPROG: Analytic
The materials price variance is computed using the
STA:
AICPA: FN-Measurement | IMA: Performance
equation
Measurement | ACBSP: APC-27-Managerial Accounting a. (Actual Price Actual Quantity) (Standard
Features/Costs
KEY:
Price Standard Quantity).
Bloom's: Application
b. (Standard Price Actual Quantity) (Actual
NOT:
3 min.
Price Actual Quantity).
c. (Standard Price Standard Quantity) (Actual
31. Refer to Figure 10-2. The variance that is higher than
Price Actual Quantity).
the upper control limit is
d. (Actual Price Actual Quantity) (Standard
a. $220,000
Price Actual Quantity).
b. $280,000
e. None of these.
c. $265,000
d. $235,000
ANS:
D
PTS:
1
DIF:
Difficulty: Moderate
ANS:
B
PTS:
1
DIF:
OBJ:
LO: 10-4
NAT:
BUSPROG: Analytic
Difficulty: Easy
STA:
AICPA: FN-Measurement | IMA: Performance
OBJ:
LO: 10-3
NAT:
BUSPROG: Analytic
Measurement | ACBSP: APC-27-Managerial Accounting
STA:
AICPA: FN-Measurement | IMA: Performance
Features/Costs
KEY:
Measurement | ACBSP: APC-27-Managerial Accounting
Bloom's: Comprehension
Features/Costs
KEY:
NOT:
3 min.
Bloom's: Application
NOT:
3 min.
The materials usage variance is calculated by the
equation
32. Refer to Figure 10-2. The variance that is lower than the a. (Standard Price Actual Quantity) (Standard
lower control limit is
Price Standard Quantity).
a. $220,000
b. (Standard Price Standard Quantity)
b. $280,000
(Standard Price Actual Quantity).
c. $265,000
c. (Actual Price Actual Quantity) (Standard
d. $235,000
Price Actual Quantity).
d. (Actual Price Standard Quantity) (Actual
Quantity Standard Price).
ANS:
A
PTS:
1
DIF:
e. None of these.
Difficulty: Easy
OBJ:
LO: 10-3
NAT:
BUSPROG: Analytic
STA:
AICPA: FN-Measurement | IMA: Performance
ANS:
A
PTS:
1
DIF:
Measurement | ACBSP: APC-27-Managerial Accounting Difficulty: Moderate
Features/Costs
KEY:
OBJ:
LO: 10-4
NAT:
BUSPROG: Analytic
Bloom's: Application
STA:
AICPA: FN-Measurement | IMA: Performance
NOT:
3 min.
Measurement | ACBSP: APC-27-Managerial Accounting
Features/Costs
KEY:
Bloom's: Comprehension

NOT:
36.

3 min.

Which of the following is not true regarding the use of


materials variance information?
a. The purchasing agent has the responsibility for
controlling the materials price variance.
b. The production manager is generally responsible
for materials usage.
c. The production manager is concerned with
minimizing scrap, waste, and rework.
d. The purchasing department is responsible for
acquiring quality materials.
e. All of these are true.

Measurement | ACBSP: APC-27-Managerial Accounting


Features/Costs
KEY:
Bloom's: Application
NOT:
3 min.
During June, Cisco Company produced 12,000 chainsaw
blades. The standard quantity of material allowed per
unit was 1.5 pounds of steel per blade at a standard
cost of $8 per pound. Cisco determined that it had a
favorable materials usage variance of $1,000 for June.
Calculate the actual quantity of materials Cisco used.
a. 17,875 pounds
b. 12,125 pounds
c. 11,875 pounds
d. 18,125 pounds

ANS:
E
PTS:
1
DIF:
Difficulty: Moderate
OBJ:
LO: 10-4
NAT:
BUSPROG: Analytic
ANS:
A
STA:
AICPA: FN-Measurement | IMA: Performance
12,000 blades 1.5 pounds = 18,000 pounds of steel
Measurement | ACBSP: APC-27-Managerial Accounting
Features/Costs
KEY:
MUV
= (AQ SQ)SP
Bloom's: Comprehension
$1,000
= (AQ 18,000)$8
NOT:
3 min.
AQ
= 17,875 pounds
During the month of March, Baker's Express purchased Or 12,000 blades 1.5 pounds $8 = $144,000
10,000 pounds of flour at $1 per pound. At the end of
$1,000 = $143,000/$8 = 17,875 pounds
March, Baker's Express found that it had an unfavorable
materials price variance of $500. The standard cost per PTS:
1
DIF:
Difficulty: Moderate
pound must be
OBJ:
LO: 10-4
a. $1.95
NAT:
BUSPROG: Analytic
b. $1.00
STA:
AICPA: FN-Measurement | IMA: Performance
c. $1.05
Measurement | ACBSP: APC-27-Managerial Accounting
d. $0.95
Features/Costs
KEY:
Bloom's: Application
NOT:
3 min.
ANS:
D
MPV
= (AP SP)AQ
During June, Cisco Company produced 12,000 chainsaw
$500
= ($1 SP)10,000
blades. The standard quantity of material allowed per
SP
= $0.95
unit was 1.5 pounds of steel per blade at a standard
cost of $8 per pound. The actual cost was $7 per pound.
The actual pounds of steel that Cisco purchased were
19,500 pounds. All materials purchased were used.
PTS:
1
DIF:
Difficulty: Moderate
OBJ:
LO: 10-4
Calculate Cisco's materials usage variance.
a. $10,500 U
NAT:
BUSPROG: Analytic
STA:
AICPA: FN-Measurement | IMA: Performance
b. $12,000 F
Measurement | ACBSP: APC-27-Managerial Accounting c. $12,000 U
Features/Costs
KEY:
d. $10,500 F
Bloom's: Application
NOT:
3 min.
ANS:
C
38. During the month of March, Baker's Express purchased 12,000 blades 1.5 pounds = 18,000 pounds of steel
10,000 pounds of flour at $1 per pound. At the end of
March, Baker's Express found that it had a favorable
MUV
= (AQ SQ)SP
materials price variance of $500. The standard cost per
= (19,500 18,000)$8
pound must be
= $12,000 U
a. $0.95
b. $1.00
c. $1.05
PTS:
1
DIF:
Difficulty: Easy
d. $1.95
OBJ:
LO: 10-4
NAT:
BUSPROG: Analytic
STA:
AICPA: FN-Measurement | IMA: Performance
ANS:
C
Measurement | ACBSP: APC-27-Managerial Accounting
MPV
= (AP SP)AQ
Features/Costs
KEY:
$500
= ($1 SP)10,000
Bloom's: Application
SP
= $1.05
NOT:
3 min.
37.

Or (10,000 $1.00) + 500 = $10,500


$10,500/10,000 = $1.05
PTS:
OBJ:
NAT:
STA:

1
DIF:
Difficulty: Moderate
LO: 10-4
BUSPROG: Analytic
AICPA: FN-Measurement | IMA: Performance

Perfect Builders makes all sorts of moldings. Its


standard quantity of material allowed is 1 foot of wood
per 1 foot of molding at a standard price of $2.00 per
foot. During August, it purchased 500,000 feet of wood
at a cost of $1.90 per foot, which produced only
499,000 feet of molding. Calculate the materials price
variance and the materials usage variance, respectively.

a.
b.
c.
d.

$50,000
$49,900
$50,000
$49,900

ANS:
MPV
MUV

F and $2,000 U
U and $2,000 F
F and $1,900 U
F and $1,900 U

A
=
=
=
=
=
=

output
Actual output

Roberts Company reports its material price variances at


the time of purchase. What is the material usage
variance for Roberts Company?
a. $900 F
b. $1,950 F
c. $2,850 F
d. $900 U

(AP SP)AQ
($1.90 $2.00)500,000
$50,000 F
(AQ SQ)SP
(499,000 500,000)$2.00
$2,000 U

ANS:
A
(2,950 3,000) $18 = $900 F

PTS:
1
DIF:
Difficulty: Moderate
OBJ:
LO: 10-4
NAT:
BUSPROG: Analytic
STA:
AICPA: FN-Measurement | IMA: Performance
Measurement | ACBSP: APC-27-Managerial Accounting
Features/Costs
KEY:
Bloom's: Application
NOT:
4 min.
42.

1,000 units

PTS:
1
DIF:
Difficulty: Moderate
OBJ:
LO: 10-4
NAT:
BUSPROG: Analytic
STA:
AICPA: FN-Measurement | IMA: Performance
Measurement | ACBSP: APC-27-Managerial Accounting
Features/Costs
KEY:
Bloom's: Application
NOT:
3 min.

Mover Company has developed the following standards During August, 10,000 units were produced. The
for one of its products:
standard quantity of material allowed per unit was 10
pounds at a standard cost of $3 per pound. If there was
Direct materials:
7.5 pounds $8 per
an unfavorable usage variance of $18,750 for August,
pound
the actual quantity of materials used must be
Direct labor:
2 hours $12 per hour
a. 106,250 pounds.
b. 93,750 pounds.
The following activity occurred during March:
c. 31,875 pounds.
d. 23,438 pounds.
Materials purchased:
5,000 pounds costing
$42,500
Materials used:
3,600 pounds
ANS:
A
Units produced:
500 units
10,000 10 $3 = $300,000
Direct labor:
1,150 hours at
$300,000 + $18,750 = $318,750
$11.80/hour
$318,750/$3 = 106,250 pounds
The company records materials price variances at the PTS:
1
DIF:
Difficulty: Moderate
time of purchase. The variable standard cost per unit for OBJ:
LO: 10-4
materials and labor is
NAT:
BUSPROG: Analytic
a. $98.
STA:
AICPA: FN-Measurement | IMA: Performance
b. $84.
Measurement | ACBSP: APC-27-Managerial Accounting
c. $74.
Features/Costs
KEY:
d. $38.
Bloom's: Application
NOT:
3 min.
ANS:
B
Direct materials (7.5 pounds $8)
Direct labor (2 hours $12)

$60
24
$84

PTS:
1
DIF:
Difficulty: Moderate
OBJ:
LO: 10-4
NAT:
BUSPROG: Analytic
STA:
AICPA: FN-Measurement | IMA: Performance
Measurement | ACBSP: APC-27-Managerial Accounting
Features/Costs
KEY:
Bloom's: Application
NOT:
3 min.

During September, 40,000 units were produced. The


standard quantity of material allowed per unit was 5
pounds at a standard cost of $2.50 per pound. If there
was a favorable usage variance of $25,000 for
September, the actual quantity of materials used must
have been
a. 210,000 pounds.
b. 190,000 pounds.
c. 105,000 pounds.
d. 95,000 pounds.

ANS:
B
40,000 5 $2.50 = $500,000
$500,000 $25,000 = $475,000
43. Roberts Company uses a standard costing system. The $475,000/$2.50 = 190,000 pounds
following information pertains to direct materials for the
July:
PTS:
1
DIF:
Difficulty: Moderate
OBJ:
LO: 10-4
Standard price per lb.
$18.00
NAT:
BUSPROG: Analytic
Actual purchase price per lb.
$16.50
STA:
AICPA: FN-Measurement | IMA: Performance
Quantity purchased
3,100 lbs.
Measurement | ACBSP: APC-27-Managerial Accounting
Quantity used
2,950 lbs.
Features/Costs
KEY:
Standard quantity allowed for actual
3,000 lbs.
Bloom's: Application

NOT:
46.

3 min.

Max Company has developed the following standards


for one of its products.
Direct materials:
Direct labor:
Variable overhead:

15 pounds $16 per


pound
4 hours $24 per hour
4 hours $14 per hour

The following activity occurred during the month of


October:
Materials purchased:
Materials used:
Units produced:
Direct labor:

10,000 pounds costing


$170,000
7,200 pounds
500 units
2,300 hours at
$23.60/hour

The company records materials price variances at the


time of purchase. The direct materials price variance is
a. $50,000 F.
b. $50,000 U.
c. $10,000 U.
d. $10,000 F.

Leather (12 strips @ $20)

$240

Direct labor (10 hours @ $12)

$120

Total prime cost

$360

During the year Bortello produced 125 boots. Actual


leather purchased was 1,700 strips, at $16 per strip.
There were no beginning or ending inventories of
leather. Actual direct labor was 1,500 hours at $15 per
hour.
Refer to Figure 10-3. Compute the materials price
variance and the materials usage variance, respectively.
a. $9,000 F and $1,200 U
b. $9,300 U and $1,500 F
c. $6,800 F and $4,000 U
d. $6,800 U and $4,000 F
ANS:
C
MPV = (AP - SP) x AQ
($16 - $20)1,700 = $6,800 F
MUV = (AQ - SQ) x SP
(1,700 - 1,500)$20 = $4,000 U

ANS:
C
$170,000 (10,000 $16) = $10,000 U
PTS:
1
DIF:
Difficulty: Moderate
OBJ:
LO: 10-4
NAT:
BUSPROG: Analytic
STA:
AICPA: FN-Measurement | IMA: Performance
Measurement | ACBSP: APC-27-Managerial Accounting
Features/Costs
KEY:
Bloom's: Application
NOT:
3 min.
47.

All of the following are true except


a. A favorable labor efficiency variance could result
from using higher quality materials that result in
fewer inspections.
b. A favorable labor rate variance could result from
lower wage workers quitting.
c. A favorable materials price variance could result
from purchasing identical materials from another
supplier at a lower price.
d. An unfavorable materials usage variance could
result from not efficiently utilizing raw materials,
thus causing waste.
e. An unfavorable labor efficiency variance can be
caused by machine downtime, and poor quality
materials.
ANS:
B
PTS:
1
DIF:
Difficulty: Moderate
OBJ:
LO: 10-4 | LO: 10-5
NAT:
BUSPROG: Analytic
STA:
AICPA: FN-Measurement | IMA: Performance
Measurement | ACBSP: APC-27-Managerial Accounting
Features/Costs
KEY:
Bloom's: Comprehension
NOT:
2 min.
Figure 10-3.
Bortello Corporation produces high-quality leather
boots. The company has a standard cost system and
has set the following standards for materials and labor:

PTS:
1
DIF:
Difficulty: Moderate
OBJ:
LO: 10-4
NAT:
BUSPROG: Analytic
STA:
AICPA: FN-Measurement | IMA: Performance
Measurement | ACBSP: APC-27-Managerial Accounting
Features/Costs
KEY:
Bloom's: Application
NOT:
4 min.
Refer to Figure 10-3. Calculate the labor rate variance
and the labor efficiency variance, respectively.
a. $4,500 U and $3,000 U
b. $4,500 F and $3,000 F
c. $4,500 U and $3,000 F
d. $4,500 F and $3,000 U
ANS:
A
LRV = (AR - SR)AH
($15 - $12)1,500 = $4,500 U
LEV = (AH - SH)SR
(1,500 - 1,250)$12 = $3,000 U

PTS:
1
DIF:
Difficulty: Moderate
OBJ:
LO: 10-5
NAT:
BUSPROG: Analytic
STA:
AICPA: FN-Measurement | IMA: Performance
Measurement | ACBSP: APC-27-Managerial Accounting
Features/Costs
KEY:
Bloom's: Application
NOT:
3 min.
Refer to Figure 10-3. Compute the total budget
variances for materials and labor, respectively.
a. $2,800 F and $7,500 F
b. $2,800 F and $7,500 U
c. $2,800 U and $7,500 U
d. $2,800 U and $7,500 F

c. $135,000 F
d. $170,000 U
ANS:

Materials
Labor

Actual
Cost*
$27,200

Budgeted
Cost^
$30,000

Variance
2,800 F

ANS:
A
MPV= (AP-SP)AQ

22,500

15,000

7,500 U

(2.00 - $2.25) x 400,000


$100,000 F

*(1,700 x $16); (1,500 x $15)


^Budgeted Materials: (SQ x SR) x AQ = $240 x 125
= $30,000
^Budgeted Labor: (SH x SR) x AQ = $120 x 125 =
$15,000

PTS:
1
DIF:
Difficulty: Moderate
OBJ:
LO: 10-4
NAT:
BUSPROG: Analytic
STA:
AICPA: FN-Measurement | IMA: Performance
Measurement | ACBSP: APC-27-Managerial Accounting
Features/Costs
KEY:
Bloom's: Application
NOT:
3 min.

PTS:
1
DIF:
Difficulty: Moderate
OBJ:
LO: 10-4 | LO: 10-5
NAT:
BUSPROG: Analytic
STA:
AICPA: FN-Measurement | IMA: Performance
Refer to Figure 10-5. What is the materials usage
Measurement | ACBSP: APC-27-Managerial Accounting
variance?
Features/Costs
KEY:
a. $112,500 F
Bloom's: Application
b. $112,500 U
NOT:
4 min.
c. $45,000 F
d. $45,000 U
51. Refer to Figure 10-3. Compute the costs of leather and
direct labor that should have been incurred for the
production of 125 boots.
ANS:
D
a. $36,000 and $36,000
b. $46,500 and $37,500
($2.25 x 400,000) - ($2.25 x 380,000)
c. $37,200 and $20,000
$45,000 U
d. $30,000 and $15,000
ANS:
D
Materials: (SQ x SR) x AQ = $240 x 125 = $30,000
Labor: (SH x SR) x AQ = $120 x 125 = $15,000

PTS:
1
DIF:
Difficulty: Moderate
OBJ:
LO: 10-4 | LO: 10-5
NAT:
BUSPROG: Analytic
STA:
AICPA: FN-Measurement | IMA: Performance
Measurement | ACBSP: APC-27-Managerial Accounting
Features/Costs
KEY:
Bloom's: Application
NOT:
4 min.
Figure 10-5.
Seaside Company produces picture frames. During
the year 190,000 picture frames were produced.
Materials and labor standards for producing the
picture frames are as follows:

Direct materials (2 pieces of wood @


$2.25)
Direct labor (2 hours @ $10)

$4.50
$20.00

Seaside purchased and used 400,000 pieces of


wood at $2.00 each and its actual labor hours were
360,000 hours at a wage rate of $10.50.

52.

Refer to Figure 10-5. What is the materials price


variance?
a. $100,000 F
b. $112,500 U

PTS:
1
DIF:
Difficulty: Moderate
OBJ:
LO: 10-4
NAT:
BUSPROG: Analytic
STA:
AICPA: FN-Measurement | IMA: Performance
Measurement | ACBSP: APC-27-Managerial Accounting
Features/Costs
KEY:
Bloom's: Application
NOT:
3 min.
Refer to Figure 10-5. What is Seaside's labor rate
variance?
a. $180,000 F
b. $180,000 U
c. $225,000 U
d. $217,500 F
ANS:
B
LRV = (AR - SR)AH
($10.50 - $10.00) x 360,000
$180,000 U

PTS:
1
DIF:
Difficulty: Moderate
OBJ:
LO: 10-5
NAT:
BUSPROG: Analytic
STA:
AICPA: FN-Measurement | IMA: Performance
Measurement | ACBSP: APC-27-Managerial Accounting
Features/Costs
KEY:
Bloom's: Application
NOT:
3 min.
Refer to Figure 10-5. What is Seaside's total labor
variance?
a. $20,000 F
b. $20,000 U
c. $112,500 F

d. $120,000 U
ANS:
A
(360,000 x $10.50) - (380,000 x $10.00)
$20,000 F

PTS:
1
DIF:
Difficulty: Moderate
OBJ:
LO: 10-5
NAT:
BUSPROG: Analytic
STA:
AICPA: FN-Measurement | IMA: Performance
Measurement | ACBSP: APC-27-Managerial Accounting
Features/Costs
KEY:
Bloom's: Application
NOT:
3 min.
56.

Which of the following is true regarding direct labor


variances?
a. The labor efficiency variance measures the
difference between what was paid to direct
laborers and what should have been paid.
b. The labor rate and labor efficiency variances will
always add up to the total labor variance.
c. The labor rate variance measures the difference
between the labor hours that were actually used
and the labor hours that should have been used.
d. The labor rate variance measures the difference
between the labor hours that were originally
budgeted and the labor hours that should have
been used.
e. The labor rate variance measures the difference
between the labor hours that were actually used
and the labor hours that were originally
budgeted.

Actual Hours).
c. (Actual Hours Standard Rate) (Standard
Hours Standard Rate).
d. (Standard Hours Actual Rate) (Actual Hours
Actual Rate).
e. None of these.
ANS:
C
PTS:
1
DIF:
Difficulty: Moderate
OBJ:
LO: 10-5
NAT:
BUSPROG: Analytic
STA:
AICPA: FN-Measurement | IMA: Performance
Measurement | ACBSP: APC-27-Managerial Accounting
Features/Costs
KEY:
Bloom's: Comprehension
NOT:
2 min.
Which of the following is not true regarding the use of
labor variance information?
a. The actual wage rate is almost always different
from the standard rate.
b. Unexpected overtime can cause variation in the
labor rate.
c. An average wage rate is chosen as the labor rate
standard.
d. The production manager controls the use of
labor.
e. The actual wage rate is used in determining the
labor rate variance.
ANS:
A
Typically, the actual wage rate is equal to the standard
rate.
PTS:
1
DIF:
Difficulty: Moderate
OBJ:
LO: 10-5
NAT:
BUSPROG: Analytic
STA:
AICPA: FN-Measurement | IMA: Performance
Measurement | ACBSP: APC-27-Managerial Accounting
Features/Costs
KEY:
Bloom's: Comprehension
NOT:
2 min.

ANS:
B
PTS:
1
DIF:
Difficulty: Easy
OBJ:
LO: 10-5
NAT:
BUSPROG: Analytic
STA:
AICPA: FN-Measurement | IMA: Performance
Measurement | ACBSP: APC-27-Managerial Accounting
Features/Costs
KEY:
Kaizen costing involves
Bloom's: Knowledge
a. changing the standards frequently.
NOT:
2 min.
b. changing management.
c. outsourcing processes.
57. The labor rate variance is computed by
d. major ad campaigns.
a. (Actual Rate Actual Hours) (Standard Rate
Standard Hours).
b. (Standard Rate Actual Rate) (Actual Rate
ANS:
A
Actual Hours).
Thus, kaizen costing differs from traditional standard
c. (Actual Rate Standard Hours) (Standard
costing in that the standard changes frequently,
Rate Actual Hours).
reflecting continuous improvement efforts.
d. (Actual Rate Actual Hours) (Standard Rate
Actual Hours).
PTS:
1
DIF:
Difficulty: Moderate
e. None of these.
OBJ:
LO: 10-5
NAT:
BUSPROG: Analytic
STA:
AICPA: FN-Decision Modeling | IMA: Performance
ANS:
D
PTS:
1
DIF:
Measurement | ACBSP: APC-27-Managerial Accounting
Difficulty: Moderate
Features/Costs
KEY:
OBJ:
LO: 10-5
NAT:
BUSPROG: Analytic
Bloom's: Knowledge
STA:
AICPA: FN-Measurement | IMA: Performance
2 min.
Measurement | ACBSP: APC-27-Managerial Accounting NOT:
Features/Costs
KEY:
Which of the following is not true about Kaizen
Bloom's: Comprehension
Standards?
NOT:
2 min.
a. Kaizen standards are the standards used for
continuous improvement.
58. The labor efficiency variance is calculated by the
b. Kaizen standards are a currently attainable
equation
standard that reflects planned improvement.
a. (Standard Hours Actual Hours) (Actual
c. Kaizen standards are constantly changing.
Hours Standard Rate).
d. Kaizen standards are the standards used in
b. (Actual Rate Actual Hours) (Standard Rate

traditional costing systems.


ANS:
D
PTS:
1
DIF:
Difficulty: Moderate
OBJ:
LO: 10-5
NAT:
BUSPROG: Analytic
STA:
AICPA: FN-Decision Modeling | IMA: Strategic
Planning | ACBSP: APC-27-Managerial Accounting
Features/Costs
KEY:
Bloom's: Knowledge
NOT:
2 min.
62.

Claire Company uses a standard costing system. The


following information pertains to direct labor costs for
February:
Standard direct labor rate per
$15.00
hour
Actual direct labor rate per hour $13.50
Labor rate variance
$18,000 F
Actual output
1,000 units
Standard hours allowed for actual 10,000 hours
production
What is the total labor budget variance for Claire
Company?
a. $18,000 F
b. $12,000 F
c. $18,000 U
d. $12,000 U
ANS:
D
$18,000/($15.00 $13.50) = 12,000 actual hours
(12,000 $13.50) (10,000 $15) = $12,000 U
PTS:
1
DIF:
Difficulty: Moderate
OBJ:
LO: 10-5
NAT:
BUSPROG: Analytic
STA:
AICPA: FN-Measurement | IMA: Strategic
Planning | ACBSP: APC-27-Managerial Accounting
Features/Costs
KEY:
Bloom's: Application
NOT:
4 min.

63.

Claire Company uses a standard costing system. The


following information pertains to direct labor costs for
February:
Standard direct labor rate per
$15.00
hour
Actual direct labor rate per hour $13.50
Labor rate variance
$18,000 F
Actual output
1,000 units
Standard hours allowed for actual 10,000 hours
production
How many actual labor hours were worked during
February for Claire Company?
a. 10,000 hours
b. 2,000 hours
c. 1,200 hours
d. 12,000 hours
ANS:
D
$18,000/($15.00 $13.50) = 12,000 hours
PTS:
1
DIF:
Difficulty: Moderate
OBJ:
LO: 10-5
NAT:
BUSPROG: Analytic
STA:
AICPA: FN-Measurement | IMA: Strategic
Planning | ACBSP: APC-27-Managerial Accounting

Features/Costs
Bloom's: Application
NOT:
4 min.

KEY:

If the actual labor rate exceeds the standard labor rate


and the actual labor hours exceed the number of hours
allowed, the labor rate variance and labor efficiency
variance will be

a.
b.
c.
d.

Labor Rate
Variance
Favorable
Favorable
Unfavorable
Unfavorable

Labor Efficiency
Variance
Favorable
Unfavorable
Favorable
Unfavorable

ANS:
D
PTS:
1
DIF:
Difficulty: Moderate
OBJ:
LO: 10-5
NAT:
BUSPROG: Analytic
STA:
AICPA: FN-Measurement | IMA: Strategic
Planning | ACBSP: APC-27-Managerial Accounting
Features/Costs
KEY:
Bloom's: Comprehension
NOT:
2 min.
During January, 7,000 direct labor hours were worked at
a standard cost of $20 per hour. If the direct labor rate
variance for January was $17,500 favorable, the actual
cost per direct labor hour must be
a. $17.50.
b. $20.00.
c. $22.50.
d. $25.00.
ANS:
A
7,000 $20 = $140,000
$140,000 $17,500 = $122,500
$122,500/7,000 = $17.50
PTS:
1
DIF:
Difficulty: Moderate
OBJ:
LO: 10-5
NAT:
BUSPROG: Analytic
STA:
AICPA: FN-Measurement | IMA: Strategic
Planning | ACBSP: APC-27-Managerial Accounting
Features/Costs
KEY:
Bloom's: Application
NOT:
2 min.
During October, 10,000 direct labor hours were worked
at a standard cost of $10 per hour. If the direct labor
rate variance for October was $4,000 unfavorable, the
actual cost per direct labor hour must be
a. $10.40.
b. $10.00.
c. $9.60.
d. $9.20.
ANS:
A
10,000 $10 = $100,000
$100,000 + $4,000 = $104,000
$104,000/10,000 = $10.40
PTS:
1
DIF:
Difficulty: Moderate
OBJ:
LO: 10-5
NAT:
BUSPROG: Analytic
STA:
AICPA: FN-Measurement | IMA: Strategic
Planning | ACBSP: APC-27-Managerial Accounting
Features/Costs
KEY:
Bloom's: Application
NOT:
2 min.

67.

Refer to Figure 10-4. What was High Fliers' total labor


Bender Corporation produced 100 units of Product AA. variance?
The total standard and actual costs for materials and
a. $61,500 F
direct labor for the 100 units of Product AA are as
b. $76,500 F
follows:
c. $76,500 U
d. $61,500 U
Materials:
Standard
Actual
Standard: 200 pounds at
$600
$3.00 per pound
ANS:
B
Actual:
220 pounds at
$627
$2.85 per pound
LEV= (AH - SH)SR
Direct labor:
Standard:
Actual:

(14,000 - 20,000) x $11


400 hours at
$15.00 per hour
368 hours at
$16.50 per hour

6,000

LEV = $66,000 F
6,072

What is the labor efficiency variance for Bender


Corporation?
a. $480 U
b. $552 F
c. $552 U
d. $480 F
ANS:
D
(368 400) $15 = $480 F
PTS:
1
DIF:
Difficulty: Easy
OBJ:
LO: 10-5
NAT:
BUSPROG: Analytic
STA:
AICPA: FN-Measurement | IMA: Strategic
Planning | ACBSP: APC-27-Managerial Accounting
Features/Costs
KEY:
Bloom's: Application
NOT:
3 min.

Total variance = $66,000 + 10,500 = $76,500

PTS:
1
DIF:
Difficulty: Moderate
OBJ:
LO: 10-5
NAT:
BUSPROG: Analytic
STA:
AICPA: FN-Measurement | IMA: Performance
Measurement | ACBSP: APC-27-Managerial Accounting
Features/Costs
KEY:
Bloom's: Application
NOT:
3 min.
Figure 10-6.
Extreme Builders constructs houses. The standard labor
rate is $25 per hour and the standard number of hours
is 15,000 hours per home. During the year, it
constructed 12 homes using 18,000 labor hours per
home and a rate of $28 per hour.

Refer to Figure 10-6. Calculate the Extreme Builders'


labor rate variance.
a. $540,000 U
Figure 10-4.
High Fliers Company produces model airplanes. During b. $540,000 F
the month of November, it produced 2,000 planes. The c. $648,000 U
actual labor hours were 7 hours per plane. Its standard d. $648,000 F
labor hours are 10 hours per plane. The standard labor
rate is $11 per hour. At the end of November, High
C
Fliers found that it had a favorable labor rate variance ANS:
LRV= (AR - SR)AH
of $10,500.
68.

Refer to Figure 10-4. What was High Fliers' actual cost


per labor hour?
a. $12.75
b. $11.50
c. $10.50
d. $10.25
ANS:

LRV = (AR - SR)AH


(10,500) = (AR - $11.00)(2,000 x 7)
AR = $10.25

PTS:
1
DIF:
Difficulty: Moderate
OBJ:
LO: 10-5
NAT:
BUSPROG: Analytic
STA:
AICPA: FN-Measurement | IMA: Performance
Measurement | ACBSP: APC-27-Managerial Accounting
Features/Costs
KEY:
Bloom's: Application
NOT:
3 min.

($28 - $25)216,000
$648,000 U

PTS:
1
DIF:
Difficulty: Easy
OBJ:
LO: 10-5
NAT:
BUSPROG: Analytic
STA:
AICPA: FN-Measurement | IMA: Performance
Measurement | ACBSP: APC-27-Managerial Accounting
Features/Costs
KEY:
Bloom's: Application
NOT:
3 min.
Refer to Figure 10-6. Calculate the labor efficiency
variance.
a. $1,008,000 F
b. $900,000 U
c. $1,008,000 U
d. $900,000 F
ANS:
B
LEV = (AH - SH)SR
(216,000 - 180,000) x $25

$900,000 U

PTS:
1
DIF:
Difficulty: Easy
OBJ:
LO: 10-5
NAT:
BUSPROG: Analytic
STA:
AICPA: FN-Measurement | IMA: Performance
Measurement | ACBSP: APC-27-Managerial Accounting
Features/Costs
KEY:
Bloom's: Application
NOT:
3 min.
72.

Assume that SQ = Standard Quantity, SP = Standard


Price, AQ = Actual Quantity, and AP = Actual Price. The
correct entry along with the equation to record the
issuance and usage of materials, assuming a favorable
materials usage variance, is as follows
a. Work in Process
SQ SP
Materials Usage Variance
(AQ
SQ)SP
Materials
AQ SP
b. Work in Process
SQ SP
Materials Usage Variance (AQ SQ)SP
Materials
AQ SP
c. Work in Process
AQ AP
Materials Usage Variance
(AQ
SQ)SP
Materials
AQ SP
d. Work in Process
AQ AP
Materials Usage Variance (AQ SQ)SP
Materials
AQ SP
e. None of these.
ANS:
A
Only standard quantities and standard prices are used
to assign costs to Work in Process.
PTS:
1
DIF:
Difficulty: Moderate
OBJ:
LO: 10-6
NAT:
BUSPROG: Analytic
STA:
AICPA: FN-Measurement | IMA: Strategic
Planning | ACBSP: APC-06-Recording Transactions
KEY:
Bloom's: Comprehension
NOT:
3 min.

73.

Which of the following is true regarding the disposition


of materials and labor variances?
a. The variances for materials and labor are closed
directly to Cost of Goods Sold regardless of
materiality.
b. If the materials price variance is material, it is
prorated among Materials Inventory, Materials
Usage Variance, Work in Process, and Finished
Goods.
c. The materials usage variance and the labor
variances, if material, are prorated among Work
in Process, Finished Goods, and Cost of Goods
Sold.
d. The materials usage variance and the labor
variances are always prorated among Work in
Process, Finished Goods, and Cost of Goods Sold.
e. The materials usage variance and the labor
variances are always closed to Cost of Goods
Sold.

If the materials price variance is material, it is prorated


among Materials Inventory, Materials Usage Variance,
Work in Process, Finished Goods, and Cost of Goods
Sold.
PTS:
1
DIF:
Difficulty: Moderate
OBJ:
LO: 10-6
NAT:
BUSPROG: Analytic
STA:
AICPA: FN-Measurement | IMA: Performance
Measurement | ACBSP: APC-27-Managerial Accounting
Features/Costs
KEY:
Bloom's: Comprehension
NOT:
3 min.
During September, a small roofing company purchased
500 bundles of a certain type of shingle at a price of
$35 per bundle, $5 less than the standard price. Its
standard quantity of this type of shingle is 550 bundles.
What is the journal entry to record the purchase of
materials?
a. Materials
20,000
Materials Price Variance
2,500
Accounts Payable
17,500
b. Materials
20,000
Materials Price Variance
2,500
Accounts Payable
22,500
c. Materials
17,500
Materials Price Variance
2,500
Accounts Payable
15,000
d. Materials
20,000
Materials Price Variance
2,750
Accounts Payable
17,250
ANS:
A
Materials = SP AQ = $40 500 = $20,000
MPV = (AP SP)AQ = ($35 $40)500 = $2,500 F
PTS:
OBJ:
NAT:
STA:
ACBSP:
KEY:
4 min.

1
DIF:
Difficulty: Moderate
LO: 10-6
BUSPROG: Analytic
AICPA: FN-Measurement | IMA: Reporting |
APC-06-Recording Transactions
Bloom's: Application
NOT:

During June, Cisco Company produced 15,000 chainsaw


blades. The standard quantity of material allowed per
unit was 1.5 pounds of steel per blade at a standard
cost of $5 per pound. The actual purchase price was
$6.25 per pound. Cisco determined that it had a
favorable materials usage variance of $2,500 for June.
What is the journal entry to record the issuance and
usage of materials?
a. Work in Process
112,500
Materials Usage Variance
2,500
Materials
110,000
b. Work in Process
110,000
Materials Usage Variance
2,500
Materials
112,000
c. Work in Process
112,500
Materials Usage Variance
2,500
Materials
115,000
d. Work in Process
140,625
Materials Usage Variance
2,500
Materials
138,125

ANS:
C
ANS:
A
The variances for materials and labor are closed directly
15,000 blades 1.5 pounds = 22,500 pounds of steel
to Cost of Goods Sold only if immaterial.
MUV
= (AQ SQ)SP
$2,500
= (AQ 22,500)$5
AQ
= 22,000 pounds

OBJ:
LO: 10-6
NAT:
BUSPROG: Analytic
STA:
AICPA: FN-Measurement | IMA: Performance
Measurement | ACBSP: APC-06-Recording Transactions
Work in Process = SQ SP = 22,500 $5 = $112,500 KEY:
Bloom's: Application
NOT:
Materials = AQ SP = 22,000 $5 = $110,000
4 min.
Or, 15,000 blades 1.5 pounds $5 = $112,500
$2,500 = $110,000/$5 = 22,000 pounds

PTS:
OBJ:
NAT:
STA:
ACBSP:
KEY:
4 min.

1
DIF:
Difficulty: Moderate
LO: 10-6
BUSPROG: Analytic
AICPA: FN-Measurement | IMA: Reporting |
APC-06-Recording Transactions
Bloom's: Application
NOT:

Figure 10-8.
The Perfect Tool Company (South America Division)
produced 80,000 saw blades during the year. It took 1.5
hours of labor per blade at a rate of $8.50 per hour.
However, its standard labor rate is $8.00. Its labor
efficiency variance was an unfavorable $40,000.

Refer to Figure 10-8. What is Perfect's standard hours


allowed for a volume of 80,000 blades?
Figure 10-7.
During April, a small roofing company purchased 700 a. 210,000 hours
bundles of a certain type of shingle at a price of $35 perb. 189,000 hours
bundle, $8 more than the standard price. Its standard c. 115,000 hours
d. 125,000 hours
quantity of this type of shingle is 725 bundles.
76.

Refer to Figure 10-7. What is the journal entry to record


the issuance and usage of materials assuming that the
roofing company purchased and used 700 bundles?
a. Work in Process
18,900
Materials Usage Variance
675
Materials
19,575
b. Work in Process
19,575
Materials Usage Variance
675
Materials
18,900
c. Work in Process
24,500
Materials Usage Variance
875
Materials
25,375
d. Work in Process
25,375
Materials Usage Variance
875
Materials
24,500
ANS:
B
Work in Process = SQ SP = 725 27 = $19,575
MUV = (AQ SQ)SP = (700 725)27 = $675 F
Materials = AQ SP = 700 $27 = $18,900

ANS:
C
LEV = (AH - SH) x SR
$40,000 = (120,000 - SH) x $8
115,000 hours

PTS:
1
DIF:
Difficulty: Moderate
OBJ:
LO: 10-5
NAT:
BUSPROG: Analytic
STA:
AICPA: FN-Measurement | IMA: Performance
Measurement | ACBSP: APC-27-Managerial Accounting
Features/Costs
KEY:
Bloom's: Application
NOT:
3 min.
Refer to Figure 10-8. What is Perfect's labor rate
variance?
a. $57,500 U
b. $57,500 F
c. $60,000 U
d. $60,000 F

PTS:
1
DIF:
Difficulty: Moderate
OBJ:
LO: 10-6
NAT:
BUSPROG: Analytic
STA:
AICPA: FN-Measurement | IMA: Performance
C
Measurement | ACBSP: APC-06-Recording Transactions ANS:
=
(AR
SR)AH
KEY:
Bloom's: Application
NOT:
4 min.
($8.50 - $8) x 120,000
77.

Refer to Figure 10-7. What is the journal entry to record $60,000 U


the purchase of materials?
a. Materials
19,575
Materials Price Variance
5,800
PTS:
1
DIF:
Difficulty: Moderate
Accounts Payable
25,375
OBJ:
LO: 10-5
b. Materials
24,500
NAT:
BUSPROG: Analytic
Materials Price Variance
5,600
STA:
AICPA: FN-Measurement | IMA: Performance
Accounts Payable
18,900
Measurement | ACBSP: APC-27-Managerial Accounting
c. Materials
18,900
Features/Costs
KEY:
Materials Price Variance
5,600
Bloom's: Application
Accounts Payable
24,500
NOT:
3 min.
d. Materials
25,375
Materials Price Variance
5,800
Refer to Figure 10-8. What is the journal entry to record
Accounts Payable
19,575
both labor variances?
a. Work In Process
920,000
Labor Rate Variance
60,000
ANS:
C
Labor Efficiency Variance
40.000
Accounts Payable = AP AQ = $35 700 = $24,500
Accrued Payroll
1,020,000
MPV = (AP SP)AQ = ($8)700 = $5,600 U
b. Work In Process
960,000
Materials = AQ SP = 700 $27 = $18,900
Labor Rate Variance
57,500
Labor Efficiency Variance
40,000
PTS:
1
DIF:
Difficulty: Moderate

Accrued Payroll
1,057,500
c. Work In Process
920,000
Labor Rate Variance
60,000
Labor Efficiency Variance
40,000
Accrued Payroll
940,000
d. Work In Process
960,000
Labor Rate Variance
57,500
Labor Efficiency Variance
40,000
Accrued Payroll
977,500

Measurement | ACBSP: APC-27-Managerial Accounting


Features/Costs
KEY:
Bloom's: Application
NOT:
3 min.
Refer to Figure 10-9. What is James materials usage
variance?
a. $250,000 F
b. $300,000 F
c. $300,000 U
d. $250,000 U

ANS:
A
= (AR - SR)AH
ANS:

($8.50 - $8) x 120,000

$60,000 U

= (AQ - SQ)SP

Accrued Payroll = (80,000 x 1.5) x $8.50 =


1,020,000

(1,400,000 - 1,500,000) x $3
$300,0000 F

Work in Process = SH x SR = 115,000 x $8 =


$920,000

SQ = 1,500,000 = 250,000 x 6

LEV = (AH - SH) x SR


$40,000 = (120,000 - SH) x $8
115,000 hours

PTS:
1
DIF:
Difficulty: Moderate
OBJ:
LO: 10-6
NAT:
BUSPROG: Analytic
STA:
AICPA: FN-Measurement | IMA: Performance
Measurement | ACBSP: APC-06-Recording Transactions
KEY:
Bloom's: Application
NOT:
4 min.

PTS:
1
DIF:
Difficulty: Easy
OBJ:
LO: 10-4
NAT:
BUSPROG: Analytic
STA:
AICPA: FN-Measurement | IMA: Performance
Measurement | ACBSP: APC-27-Managerial Accounting
Features/Costs
KEY:
Bloom's: Application
NOT:
3 min.
Refer to Figure 10-9. What is James labor rate variance?
a. $190,625 F
b. $250,000 F
c. $250,000 U
d. $193,750 U

Figure 10-9.
ANS:
C
James Company manufactures t-shirts. During the year, (AR - SR)AH
it manufactured 250,000 t-shirts, using 2 hours of direct
($8.50 - $8.00) x 500,000
labor at a rate of $8.50 per hour. The materials and
labor standards for manufacturing the t-shirts are:
$250,000 U
Direct materials (6 yards of fabric @ $3 per
yard)
Direct labor (2.4 hours @ $8.00 per hour)

$18
17

It took James 1,400,000 yards at $2.50 per yard to


make the 250,000 t-shirts.
81.

Refer to Figure 10-9. What is James materials price


variance assuming that materials purchased equals
materials used?
a. $750,000 F
b. $700,000 F
c. $700,000 U
d. $750,00 U
ANS:
B
MPV = (AP - SP) x AQ
($2.50 - $3.00) x 1,400,000

AH = 500,000 = 250,000 x 2

PTS:
1
DIF:
Difficulty: Easy
OBJ:
LO: 10-5
NAT:
BUSPROG: Analytic
STA:
AICPA: FN-Measurement | IMA: Performance
Measurement | ACBSP: APC-27-Managerial Accounting
Features/Costs
KEY:
Bloom's: Application
NOT:
3 min.
Refer to Figure 10-9. What is James labor efficiency
variance?
a. $800,000 U
b. $850,000 F
c. $800,000 F
d. $850,000 U

$700,000 F
ANS:
C
(AH - SH)SR
PTS:
OBJ:
NAT:
STA:

1
DIF:
Difficulty: Easy
LO: 10-4
BUSPROG: Analytic
AICPA: FN-Measurement | IMA: Performance

(500,000 - 600,000) x $8
$800,000 F

OBJ:
LO: 10-6
NAT:
BUSPROG: Analytic
STA:
AICPA: FN-Measurement | IMA: Performance
Measurement | ACBSP: APC-06-Recording Transactions
KEY:
Bloom's: Application
NOT:
3 min.

AH = 500,000 = 250,000 x 2
SH = 600,000 = 250,000 x 2.4

PTS:
1
DIF:
Difficulty: Easy
Refer to Figure 10-9. What is the entry to close the
OBJ:
LO: 10-5
variances of labor and materials?
NAT:
BUSPROG: Analytic
a. Materials Price Variance
700,000
STA:
AICPA: FN-Measurement | IMA: Performance
Materials Usage Variance
300,000
Measurement | ACBSP: APC-27-Managerial Accounting
Labor Efficiency Variance
800,000
Features/Costs
KEY:
Cost of Goods Sold
1,800,000
Bloom's: Application
NOT:
3 min.
Cost of Goods Sold
250,000
Labor Rate Variance
250,000
85. Refer to Figure 10-9. What is the entry to record the
b. Materials Price Variance
750,000
purchase of materials?
Materials Usage Variance
250,000
a. Materials
3,500,000
Labor Efficiency Variance
850,000
Materials Price Variance 750,000
Cost of Goods Sold
1,850,000
Accounts Payable
4,250,000
b. Materials
4,200,000
Cost of Goods Sold
300,000
Materials Price Variance 750,000
Labor Rate Variance
300,000
Accounts Payable
4,950,000
c. Cost of Goods Sold
1,800,000
c. Materials
4,200,000
Materials Price Variance
700,000
Materials Price Variance
700,000
Materials Usage Variance
300,000
Accounts Payable
3,500,000
Labor Efficiency Variance
800,000
d. Materials
3,500,000
Materials Price Variance
700,000
Labor Rate Variance
250,000
Accounts Payable
4,200,000
Cost of Goods Sold
250,000
ANS:
C
Materials = SP x AQ = $3 x 1,400,000 = $4,200,000
Accounts Payable = 1,400,000 x $2.50 =
$3,500,000
Material Price Variance = ($2.50 - $3.00) x
1,400,000 = $700,000 F
PTS:
1
DIF:
Difficulty: Easy
OBJ:
LO: 10-6
NAT:
BUSPROG: Analytic
STA:
AICPA: FN-Measurement | IMA: Performance
Measurement | ACBSP: APC-06-Recording Transactions
KEY:
Bloom's: Application
NOT:
3 min.
86.

d. Cost of Goods Sold


1,850,000
Materials Price Variance
750,000
Materials Usage Variance
250,000
Labor Efficiency Variance
850,000
Labor Rate Variance
Cost of Goods Sold

300,000
300,000

ANS:
A
Labor Efficiency Variance = (AH - SH)SR = (500,000
- 600,000)$8 = $800,000 F
Material Usage Variance = (AQ - SQ)SP =
(1,400,000 - 1,500,000)$3 = $300,000 F
Material Price Variance = (AP - SP)AQ = ($2.50 $3.00)1,4000,000= $700,000 F
LRV = (AR - SR)AH = ($8.50 - $8.00)500,000 =
$250,000 U

Refer to Figure 10-9. What is the entry to record the


PTS:
1
DIF:
Difficulty: Challenging
issuance and usage of materials?
OBJ:
LO: 10-6
a. Work in Process
4,500,000
NAT:
BUSPROG: Analytic
Materials Usage Variance
300,000
STA:
AICPA: FN-Measurement | IMA: Performance
Materials
4,800,000
Measurement | ACBSP: APC-06-Recording Transactions
b. Work in Process
4,200,000
KEY:
Bloom's: Application
NOT:
Materials Usage Variance
300,000
4 min.
Materials
4,500,000
c. Work in Process
4,200,000
PROBLEM
Materials Usage Variance
300,000
Materials
3,900,000
Top Notch Music Inc. produces car stereos. During the
d. Work in Process
4,500,000
year Top Notch Music produced 7,000 stereos. Materials
Materials Usage Variance
300,000
and labor standards for producing these units are as
Materials
4,200,000
follows:
ANS:
D
Materials = SP x AQ = $3 x 1,400,000 = $4,200,000
Work in Process = SQ x SP = (250,000 x 6) x $3 =
$4,500,000
Material Usage Variance = (1,400,000 - (250,000 x 6))
$3 = $300,000 F
PTS:

DIF:

Difficulty: Easy

Direct materials (1 electronic component


kit @ $185)
Direct materials (2 plastic casing @ $45)

185

Direct labor (8 hours @ $15)

120

Required:

90

A. Compute the standards hours allowed for a


volume of 7,000 stereos and the planned cost.
B. Compute the standard number of kits and
casings allowed for a value of 7,000 units and the
planned cost for each direct material.
C. Compute the total budget variances for materials
and labor assuming that actual number of
electronic kits purchased and used were 7,300 at a
price of $179 and actual plastic casings purchased
were 14,400 at a price of $43. Actual labor was
57,200 hours at $15.75 per hour.

ANS:
A.
SH = 8 hours x 7,000 = 56,000

B.
SQ = 2 x 7,000 = 14,000 plastic casings
PC = 7,000 x $185 = $1,295,000 electronic kits
PC = 14,000 x $45 = $630,000 plastic casings

C.

PTS:
1
DIF:
Difficulty: Moderate
OBJ:
LO: 10-3 | LO: 10-4
NAT:
BUSPROG: Analytic
STA:
AICPA: FN-Measurement | IMA: Performance
Measurement | ACBSP: APC-33-Incremental analysis
KEY:
Bloom's: Application
NOT:
5 min.

During June, the company purchased and used 9,500


pounds of concrete for $5 per pound. It was able to
make 20,000 bricks. Its standard quantity of materials
allowed is 0.45 pound of concrete per brick at a
standard price of $6 per pound.
A.

Actual Budgeted Variance


Cost
Cost
1,306,
1,295, 11,70U
700
000
0
619,
200
900,
900

Standard Labor Cost = 475 $14.50 =


$6,887.50 10% = $688.75.
The unfavorable LEV of $362.50 is greater than
the $100 rule but not the 10% rule. Hence, the
variance should be investigated.

A materials usage variance will be investigated when


the variance is greater than either $5,000 or 10% of the
standard cost.

SQ = 1 x 7,000 = 7,000 electronic kits

630,
000
840,
000

(10,80F
0)
60,90U
0

PTS:
1
DIF:
Difficulty: Moderate
OBJ:
LO: 10-2 | LO: 10-3 | LO: 10-4
NAT:
BUSPROG: Analytic
STA:
AICPA: FN-Measurement | IMA: Performance
Measurement | ACBSP: APC-33-Incremental analysis
KEY:
Bloom's: Application
NOT:
10 min.

2.

B.

Acme Brick Company uses the following rule to


determine whether materials usage variances should be
investigated:

PC = $15 x 56,000 = $840,000

Materials
(electronic
component kits)
Materials (plastic
casings)
Labor

= (500 475)$14.50
= $362.50 U

B.

Determine Acme's material usage variance


and whether it is favorable or unfavorable.
Should the variance be investigated?

ANS:
= (AQ SQ)SP
= (9,500 9,000)$6
= $3,000 U

A.

MUV

B.

Standard Materials Cost = 9,000 $6 =


$54,000 10% = $5,400.
The unfavorable MUV of $3,000 is not greater
than the $5,000 rule or greater than 10% of
the standard cost. Hence, the variance should
not be investigated.

PTS:
1
DIF:
Difficulty: Easy
OBJ:
LO: 10-3 | LO: 10-4
NAT:
BUSPROG: Analytic
STA:
AICPA: FN-Measurement | IMA: Performance
Measurement | ACBSP: APC-33-Incremental analysis
Bloom's: Application
NOT:
A labor efficiency variance will be investigated when the KEY:
5 min.
variance is greater than either $100 or 10% of the
standard labor cost.
Westminster Company has the following information
During September, the company used 500 direct labor concerning its direct materials:
hours at a rate of $15 per hour. Its standard rate is 475
Direct Materials:
direct labor hours at a rate of $14.50 per hour.
Standard Quantity
100,000
Actual Quantity
80,000
A.
Determine the company's labor efficiency
Standard Price
$3
variance and whether it is favorable or
Actual Price
$4
unfavorable.
B.
Should the variance be investigated?
A.
Determine the materials price variance and
whether it is favorable or unfavorable.
B.
Determine the materials usage variance and
ANS:
whether it is favorable or unfavorable.
C.
Westminster has set control limits stating that
A.
LEV
= (AH SH)SR
Leeds Company uses the following rule to determine
whether labor efficiency variances should be
investigated:

actual costs should be investigated if they fall


outside the acceptable range of the standard
materials cost 10%.
i.
What is the standard materials cost?
ii.
What are the upper and lower control
limits?
iii. What is the actual materials cost?
iv. Should the actual materials cost be
investigated?

B.
MUV = (AQ - SQ) x SP
(7,320,000 - 7,200,000) x $0.08 = $9,600 U

PTS:
1
DIF:
Difficulty: Easy
OBJ:
LO: 10-3 | LO: 10-4
NAT:
BUSPROG: Analytic
STA:
AICPA: FN-Measurement | IMA: Performance
Measurement | ACBSP: APC-27-Managerial Accounting
Features/Costs | ACBSP: APC-33-Incremental analysis
KEY:
Bloom's: Application
NOT:
5 min.

ANS:
A.

MPV

= (AP SP)AQ
= ($4 $3)80,000
= $80,000 U

B.

MUV

= (AQ SQ)SP
= (80,000 100,000)$3
= $60,000 F

C.

i.
ii.

iii.
iv.

5.

McDaniel Company manufactures 100-pound bags of


fertilizer that have the following unit standard costs for
direct materials and direct labor:

The standard materials cost = 100,000


$3 = $300,000.
The upper control limit = $300,000 +
($300,000 10%) = $330,000.
The lower control limit = $300,000
($300,000 10%) = $270,000.
The actual materials cost = $80,000
$4 = $320,000.
No, $320,000 falls between $270,000 and
$330,000.

Direct materials (100 lbs. @ $1.00 per


lb.)
Direct labor (0.5 hours at $24 per hour)
Total standard prime cost per 100 lb.
bag

$100.00
12.00
$112.00

The following activities were recorded for October:

1,000 bags were manufactured.


95,000 lbs. of materials costing $76,000 were
purchased.
102,500 lbs. of materials were used.
$12,000 was paid for 475 hours of direct labor.

PTS:
1
DIF:
Difficulty: Moderate
There were no beginning or ending work-in-process
OBJ:
LO: 10-3 | LO: 10-4
inventories.
NAT:
BUSPROG: Analytic
STA:
AICPA: FN-Measurement | IMA: Performance
Required:
Measurement | ACBSP: APC-33-Incremental analysis
Compute the direct materials variances.
KEY:
Bloom's: Application
NOT: A.
B.
Compute the direct labor variances.
10 min.
C.
Give possible reasons for the occurrence of
each of the preceding variances.
PURE Inc. produces flavored waters, sold in gallons.
Recently the company adopted the following materials
standard for one gallon of its raspberry flavored water:
Direct materials (90 oz. @ $0.08)

7.20

ANS:
A.

[$76,000 (95,000 $1.00)] = $19,000 F

During the first month of operations the company


experienced the following results:
A. Gallon units produced: 80,000
B. Ounces of materials purchased and used:
7,320,000 ounces at $0.07
C. No beginning or ending inventories of raw
materials
Required:
A. Compute the materials price variance indicating
if it is favorable or unfavorable.
B. Compute the materials usage variance indicating
if it is favorable or unfavorable.
ANS:
A.
MPV = (AP - SP) x AQ
($0.07 - $0.08) x 7,320,000 = $73,200 F

Material price variance:

Material usage variance


[102,500 1,000(100)] $1.00 = $2,500 U
B.

Labor rate variance


[$12,000 (475 hrs. $24)] = $600 U
Labor efficiency variance
[(475 500)]$24 = $600 F

C.

All of the material price variances could be


caused by out-of-date or inappropriate
standards. Other potential reasons could be
that the firm could be purchasing in larger
quantities (larger quantity discounts),
purchasing lower grade materials, or that the
supplier could be forced to offer a lower price
due to the economics of their product.
Material usage variance:
Low-quality materials; lower skilled workers;
less efficient machines; low employee morale.
Labor rate variance:
Higher skilled workers; longer tenured workers

with higher wages.


Labor efficiency variance:
The firm could be using a more experienced
work force than desired.
PTS:
1
DIF:
Difficulty: Challenging
OBJ:
LO: 10-3 | LO: 10-4 | LO: 10-5
NAT:
BUSPROG: Analytic
STA:
AICPA: FN-Measurement | IMA: Strategic
Planning | ACBSP: APC-33-Incremental analysis
KEY:
Bloom's: Application
NOT:
10 min.
7.

E.
LRV = (AR -SR) x AH
(19.25 - 18.00) x 120,000 = $150,000 U
F.
LEV = (AH - SH) x SR
(120,000 - 110,000) x $18 = $180,000 U

PTS:
1
DIF:
Difficulty: Moderate
DuRoss Company produces coats. The company uses a OBJ:
LO: 10-3 | LO: 10-4 | LO: 10-5
NAT:
standard costing system and has set the following
BUSPROG: Analytic
standards for materials and labor:
STA:
AICPA: FN-Measurement | IMA: Performance
Measurement | ACBSP: APC-27-Managerial Accounting
Fabric (8 yards @ $6)
$48
Features/Costs | ACBSP: APC-33-Incremental analysis
KEY:
Bloom's: Application
NOT:
Direct labor (2 hours @ $18)
$36
10 min.
Total prime cost
$84
Moving Baby Company produces baby strollers. During
the year 90,000 strollers were produced. The actual
During the year DuRoss produced 55,000 coats. Actual labor used was 225,000 hours at $12.75 per hour.
fabric purchased was 460,000 yards at $5.75 per yard. Moving Baby has the following labor standards: 2 hours
There were no beginning or ending inventories of fabric. at $13.00 per hour.
Actual direct labor was 120,000 hours at $19.25 per
hour.
Required:
Required:
A. Compute the cost of leather and direct labor that
should be incurred for the production of 55,000
coats.
B. Compute the total budget variances for materials
and labor.
C. Compute the materials price variance.

A. Compute the labor rate variance, indicating if it is


favorable or unfavorable.
B. Compute the labor efficiency variance, indicating
if it is favorable or unfavorable.
ANS:
A.

D. Compute the materials usage variance.

LRV = (AR - SR) x AH

E. Compute the labor rate variance.

($12.75 - $13.00) x 225,000 = $56,250 F

F. Compute the labor efficiency variance.


B.
LEV = (AH - SH) x SR

ANS:
A.

(225,000 - 180,000) x $13.00 = $585,000 U

Materials = $48 x 55,000 = $2,640,000


Labor = $36 x 55,000 = $1,980,000

B.

Materials
Labor

Actual Cost Budgeted Variance


Cost
$2,645,
$2,640,
$5,0U
000
000
00
$2,310,
$1,980,
$330,U
000
000
000

C.
MPV = (AP - SP) x AQ
($5.75 - $6) x 460,000 = $115,000 F
D.
MUV = (AQ - SQ) x SP
(460,000 - 440,000) x $6 = $120,000 U

PTS:
1
DIF:
Difficulty: Moderate
OBJ:
LO: 10-3 | LO: 10-5
NAT:
BUSPROG: Analytic
STA:
AICPA: FN-Measurement | IMA: Performance
Measurement | ACBSP: APC-27-Managerial Accounting
Features/Costs | ACBSP: APC-33-Incremental analysis
KEY:
Bloom's: Application
NOT:
5 min.
Starling Manufacturing has developed the following
standards for one of its products.
Materials: 5 yards $6
per yard
Direct labor: 2 hours $8
per hour

$30
16

The company records materials price variances at the


time of purchase.
The following activity occurred during December:

Materials purchased:
Materials used:
Units produced:
Direct labor:
Required:
A.
Calculate
B.
Calculate
C.
Calculate
D.
Calculate

the
the
the
the

direct
direct
direct
direct

5,200 yards costing


$29,900
4,750 yards
1,000 units
2,100 hours costing
$17,850
materials price variance.
materials usage variance.
labor rate variance.
labor efficiency variance.

KEY:
Bloom's: Application
10 min.

Crawford Corporation has the following information:


Direct Materials:
Standard
1,500
Quantity
Actual Quantity
1,400
Standard Price
$20
Actual Price
$17.50
A.

ANS:
A.
B.
C.
D.

$29,900 (5,200 $6)


$6 (4,750 5,000)
$17,850 (2,100 $8)
$8 (2,100 2,000)

PTS:
1
DIF:
Difficulty: Moderate
OBJ:
LO: 10-4 | LO: 10-5
NAT:
BUSPROG: Analytic
STA:
AICPA: FN-Measurement | IMA: Performance
Measurement | ACBSP: APC-33-Incremental analysis
KEY:
Bloom's: Application
NOT:
5 min.
10.

C.
D.

A.

Direct Labor:
Standard
2,000
Hours
Actual Hours
1,875
Standard Rate
$10
Actual Rate
$11

Determine the materials price variance and


whether it is favorable or unfavorable.
Determine the materials usage variance and
whether it is favorable or unfavorable.
Determine the labor rate variance and whether
it is favorable or unfavorable.
Determine the labor efficiency variance and
whether it is favorable or unfavorable.

B.
C.
D.

ANS:
A.

B.

MPV

MUV

= (AP SP)AQ
= ($12 $14)12,000
= $24,000 F
= (AQ SQ)SP
= (12,000 10,000)$14
= $28,000 U

C.

LRV

= (AR SR)AH
= ($11 $10)1,875
= $1,875 U

D.

LEV

= (AH SH)SR
= (1,875 2,000)$10
= $1,250 F

PTS:
1
DIF:
Difficulty: Moderate
OBJ:
LO: 10-4 | LO: 10-5
NAT:
BUSPROG: Analytic
STA:
AICPA: FN-Measurement | IMA: Performance
Measurement | ACBSP: APC-33-Incremental analysis

500
525
$14
$14

ANS:
A.

MPV

= (AP SP)AQ
= ($17.50 $20)1,400
= $3,500 F

B.

MUV

= (AQ SQ)SP
= (1,400 1,500)$20
= $2,000 F

C.

LRV

= (AR SR)AH
= ($14 $14)525
= $0

D.

LEV

= (AH SH)SR
= (525 500)$14
= $350 U

Eastminster Company has the following information:


Direct Materials:
Standard
10,000
Quantity
Actual Quantity
12,000
Standard Price
$14
Actual Price
$12

Direct Labor:
Standard
Hours
Actual Hours
Standard Rate
Actual Rate

Determine the materials price variance and


whether it is favorable or unfavorable.
Determine the materials usage variance and
whether it is favorable or unfavorable.
Determine the labor rate variance and whether
it is favorable or unfavorable.
Determine the labor efficiency variance and
whether it is favorable or unfavorable.

B.

$1,300 F
$1,500 F
$1,050 U
$800 U

NOT:

PTS:
1
DIF:
Difficulty: Moderate
OBJ:
LO: 10-4 | LO: 10-5
NAT:
BUSPROG: Analytic
STA:
AICPA: FN-Measurement | IMA: Performance
Measurement | ACBSP: APC-33-Incremental analysis
KEY:
Bloom's: Application
NOT:
10 min.
Allison Company adopted a standard cost system
several years ago. The standard costs for the prime
costs of its single product follow:
Material: 10 kilograms @ $4.50 per
kilogram
Labor: 6 hours @ $8.50 per hour

$45.00
$51.00

The following operating data were taken from the


records for November:
1.
2.
3.
4.
5.
6.
7.

Units completed:
5,800 units
Budgeted output:
6,000 units
Materials purchased:
60,000 kilograms
Total actual labor costs: $306,600
Actual hours of labor:
36,500 hours
Material usage variance:$2,250 U
Total material variance: $450 U

Compute the following:


A.
B.
C.

Labor rate variance


Labor efficiency variance
Actual kilograms of material used in the

production process
Actual cost paid per kilogram of material

D.

it is favorable or unfavorable.
Determine the labor efficiency variance and
whether it is favorable or unfavorable.

D.

ANS:
A.
Standard
Cost
Labor
Rate
Variance

ANS:
Actual
Actual
Quantity A.
MPV
Cost Difference or Hours Variance

$8.50/hr. $8.40/hr.*

$.10
(fav.)

36,500
hrs.B.

MUV

* $306,600 labor cost/36,500 hours worked = $8.40 per hour


B.
Labor
Efficiency
Variance

Standard
Quantity
or Hours
34,800
hrs.**

Actual
C.
LRV
Quantity
Standard
or Hours Difference
Cost Variance
36,500 1,700 hrs. $8.50/hr. D.
hrs.
(unfav.)

LEV

** 5,800 units produced 6 hrs allowed = 34,800


C.

D.

= (AP SP)AQ
= ($2.25 $2.00)53,000
= $13,250 U
= (AQ SQ)SP
= (53,000 50,000)$2
= $6,000 U
= (AR SR)AH
= ($8.00 $7.50)21,000
= $10,500 U
= (AH SH)SR
= (21,000 20,000)$7.50
= $7,500 U

Standard
Quantity
of Kg.

Actual
PTS:
1
DIF:
Difficulty: Moderate
Quantity
Standard OBJ:
LO: 10-4 | LO: 10-5
of Kg. Difference
Cost Variance
NAT:
BUSPROG: Analytic
Materials
STA:
AICPA: FN-Measurement | IMA: Performance
Usage
58,000 unknown 500 kg.# $4.50/kg. Measurement | ACBSP: APC-33-Incremental analysis
Variance
kg.***
kg.
(unfav.)
KEY:
Bloom's: Application
NOT:
10 min.
*** 5,800 units 10 kg. per unit
# working back: $2,250/$4.50 = 500 kg.
Rhodes Corporation manufactures a product with the
The actual kg. would be 58,500 (58,000 + 500) The following standard costs:
unfavorable variance is added to standard.
Direct materials (20 yards @ $1.85 per
$37.00
Actualyard)
Standard
Actual
QuantityDirect labor (4 hours @ $12.00 per
48.00
Cost
Cost Difference
of Kg.hour)
Variance
Materials
Price
$4.50/kg. Unknown $0.03##
60,000 Standards are based on normal monthly production
Variance
(fav.)
kg. involving 2,000 direct labor hours (500 units of output).

## $1,800/60,000 = $0.03 (rounded)


The following information pertains to July:
Working back: $4.50 $.03 = $4.47 (Favorable variance is
subtracted from standard)
Direct materials purchased (16,000
$28,800
yards @ $1.80 per yard)
Direct materials used (9,400 yards)
PTS:
1
DIF:
Difficulty: Challenging Direct labor (1,880 hours @ $12.20 per
22,936
OBJ:
LO: 10-4 | LO: 10-5
hour)
NAT:
BUSPROG: Analytic
Actual production in July: 460 units
STA:
AICPA: FN-Measurement | IMA: Performance
Measurement | ACBSP: APC-33-Incremental analysis
Required:
KEY:
Bloom's: Application
NOT: A.
Compute the following variances for the month
10 min.
of July, indicating whether each variance is
favorable or unfavorable:
13. Gardener's Market manufactures hedgers. During the
year, it manufactured 5,000 hedgers, using 4.2 hours of
1.
Materials purchase price variance
direct labor per hedger at a rate of $8. The materials
2.
Materials usage variance
and labor standards for manufacturing the hedgers are:
3.
Labor rate variance
4.
Labor efficiency variance
Direct materials (10 units @ $2)
$20
Direct labor (4 hours @ $7.50 per hour)
30
B.
Give potential reasons for each of the
variances. Be sure to consider interGardener's Market actually purchased and used 53,000
relationships among variances.
units of direct materials at a price of $2.25 per unit.
Required:
A.
Determine the materials price variance and
whether it is favorable or unfavorable.
B.
Determine the materials usage variance and
whether it is favorable or unfavorable.
C.
Determine the labor rate variance and whether

ANS:
A.

Materials purchase price variance = (Actual


unit price standard unit price) actual
quantity of materials purchased
Materials purchase price variance = ($1.80

$1.85) 16,000 = $800 favorable


(actual price less than standard price)

ANS:

Materials quantity variance = (Actual quantity


of materials used standard quantity of
materials allowed) standard unit price
Materials quantity variance = (9,400
9,200*) $1.85 = $370 unfavorable
(actual quantity exceeds standard quantity)
* 460 units 20 yards per unit = 9,200
Labor rate variance = (Actual rate per hour
standard rate per hour) Actual hours worked
Labor rate variance = ($12.20 $12.00)
1,880 = $376 unfavorable
(actual rate exceeds standard rate)
Labor efficiency variance = (Actual hours
worked standard hours allowed) standard
rate
Labor efficiency variance = (1,880 1,840**)
$12.00 = $480 unfavorable
(actual hours exceed standard hours
allowed)
** 460 units 4 hours per unit = 1,840
B.

15.

The favorable purchase price variance may


have occurred because the purchasing
manager purchased materials at a lower price
that were of lesser quality. The workers
encountered production problems as a result
of the lesser quality materials which resulted
in using more materials and taking more time
than anticipated. The supervisor also had to
assign more experienced workers to this
production, which resulted in a higher average
wage rate.

A.

MPV for
Beef
MPV for
Bags
Total MPV

= (AP SP)AQ
= ($1.10 $1.00)3,300
= $330 U
= ($0.15 $0.25)3,000
= $300 F
= $30 U

B.

MUV

= (AQ SQ)SP
= (3,300 3,000)$1.00
= $300 U

C.

LRV

= (AR SR)AH
= ($9.00 $9.00)750
= $0 Neither favorable nor
unfavorable

D.

LEV

= (AH SH)SR
= (750 900)$9.00
= $1,350 F

PTS:
1
DIF:
Difficulty: Moderate
OBJ:
LO: 10-4 | LO: 10-5
NAT:
BUSPROG: Analytic
STA:
AICPA: FN-Measurement | IMA: Performance
Measurement | ACBSP: APC-33-Incremental analysis
KEY:
Bloom's: Application
NOT:
10 min.
Mersey Company produced 1,000 trash cans during
March using 450 direct labor hours and purchased and
used 3,100 pounds of rubber. Its materials and labor
standards are:

Direct materials (3 pounds of rubber @


$1.50
PTS:
1
DIF:
Difficulty: Challenging $0.50)
OBJ:
LO: 10-4 | LO: 10-5
Direct labor (0.5 hours @ $16.00)
3.00
NAT:
BUSPROG: Analytic
STA:
AICPA: FN-Measurement | IMA: Performance
Its materials price variance was a favorable $620 and
Measurement | ACBSP: APC-33-Incremental analysis
its labor rate variance was an unfavorable $900.
KEY:
Bloom's: Application
NOT:
10 min.
A.
Calculate the actual price per unit.
B.
Calculate the actual labor rate.
Dog's Best Friend manufactures dog food. During the C.
Determine the materials usage variance and
month, it manufactured 3,000 bags of kibble, using 0.25
whether it is favorable or unfavorable.
hour of direct labor per bag at a rate of $9.00 per hour. D.
Determine the labor efficiency variance and
The materials and labor standards for manufacturing
whether it is favorable or unfavorable.
the bags of kibble are:
Direct materials (1 pound of beef @
$1.00 per pound)
Direct materials (1 bag @ $0.25)
Direct labor (0.30 hour @ $9.00)

$1.00
0.25
2.70

ANS:
MPV
$620
AP

= (AP SP)AQ
= (AP $0.50)3,100
= $0.30 per pound

LRV
$900
AR

= (AR SR)AH
= (AR $16.00)450
= $18.00 per hour

C.

MUV

= (AQ SQ)SP
= (3,100 3,000)$0.50
= $50 U

D.

LEV

= (AH SH)SR
= (450 500)$16
= $800 F

A.

The company actually used 3,300 pounds of beef at a


price of $1.10 per pound. It also purchased 3,000 bags B.
at a price of $0.15 per bag.
A.
B.
C.
D.

Determine the total materials price variance


and whether it is favorable or unfavorable.
Determine the materials usage variance for
beef and whether it is favorable or
unfavorable.
Determine the labor rate variance and whether
it is favorable or unfavorable.
Determine the labor efficiency variance and
whether it is favorable or unfavorable.

PTS:

DIF:

Difficulty: Moderate

OBJ:
LO: 10-4 | LO: 10-5
NAT:
BUSPROG: Analytic
STA:
AICPA: FN-Measurement | IMA: Performance
Measurement | ACBSP: APC-33-Incremental analysis
KEY:
Bloom's: Application
NOT:
10 min.
17.

Issuance of raw materials


Addition of labor to Work in Process
Closing of variances to Cost of Goods Sold

Pontefract Company produced 2,500 widgets during


November using 4,000 units of materials at a cost of
ANS:
$5.00 each. It also used 5,000 direct labor hours at a
A.
rate of $7.00. Its direct materials standard is 2 units per
widget. Its direct labor standard is 2.5 hours per widget. MPV = (AP - SP) x AQ
($1.23 - $1.20) x 150,000 = $4,500 U
Its materials price variance was a favorable $8,000 and
its labor rate variance was an unfavorable $1,000.
A.
B.
C.

Calculate the standard materials price per unit.


Calculate the standard labor rate.
Determine the materials usage variance and
whether it is favorable or unfavorable.
Determine the labor efficiency variance and
whether it is favorable or unfavorable.

D.

A.

B.

MUV = (AQ - SQ) x SP


(150,000 - 100,000) x $1.20 = $60,000 U
B.
LRV = (AR- SR) x AH
($9.25 - $9.00) x 36,800 = $9,200 U

ANS:
MPV
$8,000
SP

= (AP SP)AQ
= ($5.00 SP)4,000
= $7.00 per pound

LEV = (AH - SH) x SR

LRV
$1,000
SR

= (AR SR)AH
= ($7.00 SR)5,000
= $6.80 per hour

C.

C.

MUV

= (AQ SQ)SP
= (4,000 5,000)$7.00
= $7,000 F

D.

LEV

= (AH SH)SR
= (5,000 6,250)$6.80
= $8,500 F

(36,800 - 37,500) x $9 = $6,300 F

Materials
Materials Price Variance

180,000
4,500

Accounts Payable

Work in Process
Materials Usage Variance

PTS:
1
DIF:
Difficulty: Challenging
OBJ:
LO: 10-4 | LO: 10-5
NAT:
BUSPROG: Analytic
STA:
AICPA: FN-Measurement | IMA: Performance
Measurement | ACBSP: APC-33-Incremental analysis
KEY:
Bloom's: Application
NOT:
10 min.
18.

Purchase of raw materials

184,500

120,000
60,000

Materials

Work in Process
Labor Rate Variance

180,000

337,500
9,200

Labor Efficiency
Just Right Inc. produces jeans. The following standards Variance
have been established:
Accrued Payroll
Direct materials (4 yards of denim @
$1.20)
Direct labor (1.5 hours @ $9)

$13.50

Standard prime cost

$18.30

6,300
340,400

$4.80

During the year 25,000 pairs of jeans were produced.


150,000 yards of denim were purchased and used at
$1.23 per yard. Actual direct labor hours were 36,800
at $9.25 per hour.
Required:
A. Compute the materials variances and indicate if
they are favorable or unfavorable.
B. Compute the labor variances and indicate if they
are favorable or unfavorable.
C. Prepare the journal entries for the following:

Cost of Goods Sold

73,700

Materials Price
Variance
Materials Usage
Variance
Labor Rate Variance
Labor Efficiency Variance
Cost of Goods Sold

PTS:
OBJ:
NAT:
STA:

4,500
60,000
9,200
6,300
6,300

1
DIF:
Difficulty: Challenging
LO: 10-3 | LO: 10-4 | LO: 10-5 | LO: 10-6
BUSPROG: Analytic
AICPA: FN-Measurement | IMA: Performance

Measurement | ACBSP: APC-06-Recording Transactions |


ACBSP: APC-27-Managerial Accounting Features/Costs |
ACBSP: APC-33-Incremental analysis
KEY:
Bloom's: Application
NOT:
15 min.
19.

Materials Price
225
Variance
Materials
500
Usage
Variance
Labor Rate
900
Variance
Cost of Goods Sold

During April, Rain Gear Unlimited produced 5,500


umbrellas from nylon that costs $0.45 per yard, which is
$0.05 cheaper than the standard cost. It also used
1,625
3,000 direct labor hours at a rate of $6.50. Its direct
materials standard is 1 yard per umbrella. Its direct
labor standard is 0.5 hour per umbrella.
PTS:
1
DIF:
Difficulty: Challenging
OBJ:
LO: 10-4 | LO: 10-5 | LO: 10-6
NAT:
Its materials usage variance was a favorable $500 and BUSPROG: Analytic
its labor rate variance was a favorable $900.
STA:
AICPA: FN-Measurement | IMA: Performance
Measurement | ACBSP: APC-06-Recording Transactions |
A.
Calculate the actual quantity of materials.
ACBSP: APC-33-Incremental analysis
KEY:
B.
Calculate the standard labor rate.
Bloom's: Application
C.
Determine the materials price variance and
NOT:
15 min.
whether it is favorable or unfavorable.
D.
Determine the labor efficiency variance and
Eider Company has the following information:
whether it is favorable or unfavorable.
E.
Record the entries for the materials purchase,
Direct Materials:
Direct Labor:
the issuance and usage of materials, and the
Standard
100,000
Standard
1,000
labor variances.
Quantity
Hours
F.
Provide the closing entries.
Actual Quantity
99,500
Actual Hours
1,050
Standard Price
$5
Standard Rate
$12
Actual Price
$4
Actual Rate
$13
ANS:
A.
Determine the materials price variance and
A.
MUV
= (AQ SQ)SP
whether it is favorable or unfavorable.
$500
= (AQ 5,500)$0.50
B.
Determine the materials usage variance and
AQ
= 4,500
whether it is favorable or unfavorable.
C.
Determine the labor rate variance and whether
B.
LRV
= (AR SR)AH
it is favorable or unfavorable.
$900
= ($6.50 SR)3,000
D.
Determine the labor efficiency variance and
SR
= $6.80 per hour
whether it is favorable or unfavorable.
E.
Provide the journal entries to record the
C.
MPV
= (AP SP)AQ
purchase of materials, the issuance and usage
= ($0.45 $0.50)4,500
of materials, and direct labor variances.
= $225 F
F.
Provide the closing entries for the immaterial
variances.
D. LEV
= (AH SH)SR
= (3,000 2,750)$6.80
= $1,700 U
ANS:
E.
Materials

2,250

Materials Price
Variance
Accounts Payable
Work in
2,750
Process
Materials Usage
Variance
Materials
Work in
Process

18,700

Labor
1,700
Efficiency
Variance
Labor Rate Variance
Accrued Payroll

(4,500
$0.50)
225
2,025

500

MPV

= (AP SP)AQ
= ($4 $5)99,500
= $99,500 F

B.

MUV

= (AQ SQ)SP
= (99,500 100,000)$5
= $2,500 F

C.

LRV

= (AR SR)AH
= ($13 $12)1,050
= $1,050 U

D.

LEV

= (AH SH)SR
= (1,050 1,000)$12
= $600 U

E.

Materials
497,500
Materials Price Variance
Accounts Payable

99,500
398,000

Work in Process
500,000
Materials Usage Variance
Materials

2,500
497,500

2,250
(0.5
5,500
$6.80)

900
19,500

F.
Cost of Goods
1,700
Sold
Labor Efficiency
Variance

A.

1,700

Work in Process
Labor Efficiency Variance
Labor Rate Variance

12,000
600
1,050

Accrued Payroll
F.

Cost of Goods Sold


Labor Efficiency Variance
Labor Rate Variance

1,650

Materials Usage Variance


Cost of Goods Sold

2,500

D.

LEV

600
1,050

E.

Materials

4,200

Materials Price
Variance
Accounts
Payable

1,235

Work in Process

4,250

PTS:
1
DIF:
Difficulty: Moderate
OBJ:
LO: 10-4 | LO: 10-5 | LO: 10-6
NAT:
BUSPROG: Analytic
STA:
AICPA: FN-Measurement | IMA: Performance
Measurement | ACBSP: APC-06-Recording Transactions |
ACBSP: APC-33-Incremental analysis
KEY:
Bloom's: Application
NOT:
15 min.

$0.50
0.35
1.40

The company actually used 4,900 pounds of tuna at a


price of $0.65 per pound. It also purchased 5,000 cans F.
at a price of $0.45 per can.
A.

Determine the total materials price variance


and whether it is favorable or unfavorable.
Determine the materials usage variance for
tuna and whether it is favorable or
unfavorable.
Determine the labor rate variance and whether
it is favorable or unfavorable.
Determine the labor efficiency variance and
whether it is favorable or unfavorable.
Make all necessary journal entries to record
the purchase of materials, the issuance and
usage of materials, and the direct labor
variances.
Provide the closing entries.

B.
C.
D.
E.

F.
ANS:
A.

MPV for
Tuna
MPV for
Cans
Total MPV

= (AP SP)AQ
= ($0.65 $0.50)4,900
= $735 U
= ($0.45 $0.35)5,000
= $500 U
= $1,235 U

B.

MUV

= (AQ SQ)SP
= (4,900 5,000)$0.50
= $50 F

C.

LRV

= (AR SR)AH
= ($8.00 $7.00)500
= $500 U

5,435

Materials
Usage
Variance
Materials

The Cat's Meow manufactures gourmet cat food. During


the month, it manufactured 5,000 cans of tuna, using
0.10 hour of direct labor per can at a rate of $8.00 per
hour. The materials and labor standards for
manufacturing the cans of tuna are as follows:
Direct materials (1 pound of tuna @
$0.50 per pound)
Direct materials (1 can @ $0.35)
Direct labor (0.20 hour @ $7.00)

(4,900
$0.50) +
(5,000
$0.35)

2,500

Since the materials price variance is material, ($99,500)


it is prorated among Materials Inventory, Materials
Usage Variance, Work in Process, Finished Goods, and
Cost of Goods Sold.

21.

= (AH SH)SR
= (500 1,000)$7.00
= $3,500 F

13,650

50

4,200(4,900
$0.50) +
(5,000
$0.35)

Work in Process
Labor Rate
Variance
Labor
Efficiency
Variance
Accrued
Payroll

7,000
500

Cost of Goods
Sold
Materials Price
Variance
Labor Rate
Variance

1,735

Materials Usage
Variance
Labor Efficiency
Variance
Cost of Goods
Sold

(5,000
$0.50) +
(5,000
$0.35)

3,500
4,000

1,235
500
50
3,500
3,550

PTS:
1
DIF:
Difficulty: Challenging
OBJ:
LO: 10-4 | LO: 10-5 | LO: 10-6
NAT:
BUSPROG: Analytic
STA:
AICPA: FN-Measurement | IMA: Performance
Measurement | ACBSP: APC-06-Recording Transactions |
ACBSP: APC-33-Incremental analysis
KEY:
Bloom's: Application
NOT:
20 min.
Grandma's Attic Company produces soft pillows made
from goose down. The company uses a standard cost
system and has set the following standards for
materials and labor for each pillow:
Feathers from 5 large white geese (5
geese @ $5)
Fabric to make pillow cases (3 yards @
$2)
Direct labor (5 hours @ $8)
Total prime cost

$25
6
40
$71

During the month, the company produced 1,000 goose


Efficiency
down pillows. Actual geese purchased were 5,100, at $4
Variance
per goose. Actual fabric purchased was 2,900 yards at
Labor Rate
1,300
$2.10 per yard. There were no beginning or ending
Variance
inventories of geese or fabric. Actual direct labor was
Accrued Payroll
40,300
5,200 hours at $7.75 per hour.
F.
Cost of Goods
1,900
A.
Determine the total materials price variance
Sold
and whether it is favorable or unfavorable.
Materials Usage
300
B.
Determine the total materials usage variance
Variance
and whether it is favorable or unfavorable.
Labor Efficiency
1,600
C.
Determine the labor rate variance and whether
Variance
it is favorable or unfavorable.
D.
Determine the labor efficiency variance and
Materials Price
4,810
whether it is favorable or unfavorable.
Variance
E.
Make all necessary journal entries to record
Labor Rate
1,300
the purchase of materials, the issuance and
Variance
usage of materials, and the direct labor
Cost of Goods Sold
6,110
variances.
F.
Provide the closing entries.
PTS:
1
DIF:
Difficulty: Challenging
OBJ:
LO: 10-4 | LO: 10-5 | LO: 10-6
NAT:
ANS:
BUSPROG: Analytic
STA:
AICPA: FN-Measurement | IMA: Performance
A.
MPV for
= (AP SP)AQ
Measurement | ACBSP: APC-06-Recording Transactions |
Geese
ACBSP: APC-33-Incremental analysis
KEY:
= ($4.00 $5.00)5,100
Bloom's: Application
= $5,100 F
NOT:
20 min.
MPV for
= ($2.10 $2.00)2,900
Fabric
Stratford Company inspects every steam iron it
= $290 U
manufactures for safety issues. The standard labor cost
is $12 per hour. The maintenance standard at the
Total MPV
= $5,100 $290 = $4,810 F
beginning of the first quarter is 20 minutes per iron.
Variance
Stratford is implementing a new production process that
will aid in reducing any potential electrical defects in
B.
MUV for
= (AQ SQ)SP
the irons. This will decrease the inspection time to 15
Geese
minutes per iron. After the end of the first quarter, the
= (5,100 5,000)$5
new process had reduced the inspection time per iron
= $500 U
from 20 minutes to 14 minutes.
MUV for
= (2,900 3,000)$2
Fabric

Total MUV

= $200 F
= $500 U $200 F = $300 U

C.

LRV

= (AR SR)AH
= ($7.75 $8.00)5,200
= $1,300 F

D.

LEV

= (AH SH)SR
= (5,200 5,000)$8.00
= $1,600 U

E.

Materials

31,300

Materials Price
Variance
Accounts Payable
Work in
Process
Materials
Usage
Variance
Materials

Work in
Process
Labor

31,000
300

1,600

B.

26,490
(5,000 $5.00)
+ (3,000
$2.00)

Identify the kaizen and maintenance labor


standards in place at the beginning of the first
quarter. Express the standards in both physical
and financial terms.
Calculate the expected cost reduction and
actual cost reduction.

ANS:
A.

(5,100 $5.00)
+ (2,900
$2.00)
4,810

31,300(5,100 $5.00)
+ (2,900
$2.00)
40,000

A.

Maintenance:
Physical standard: 20 minutes
Financial standard: $4.00 ($12 20/60 hours)
Kaizen:
Physical standard: 15 minutes
Financial standard: $3.00 ($12 15/60 hours)

B.

Expected cost reduction: $4.00 $3.00 =


$1.00 per unit
Actual cost reduction: $4.00 ($12 14/60
hours) = $1.20 per unit

PTS:
1
DIF:
Difficulty: Moderate
OBJ:
LO: 10-5
NAT:
BUSPROG: Analytic
STA:
AICPA: FN-Decision Modeling | IMA: Strategic
Planning | ACBSP: APC-27-Managerial Accounting
Features/Costs
KEY:
Bloom's: Application
NOT:
5 min.

24.

Wiltshire Limited produces woolen blankets and


clothing. During Year 1, Wiltshire produced 10,000 itemsA.
of blankets and clothing using 4,250 bundles of wool at B.
a price of $10 per bundle.
Standard bundles of wool
Standard price

4,000
$20 per bundle

C.

Calculate Overland's target cost.


Calculate the total costs per car after Overland
redesigns its processes and schedules to buy
cost-saving components.
Should Overland manufacture the car?
Calculate the expected profit after the cost
savings are taken into account.

The difference between the actual quantity of materials


and the standard quantity of materials is due to waste.
At the end of Year 1, Wiltshire developed a new process ANS:
that would cut down on the waste by 60%. By the end
of Year 2, the company had actually cut down its waste A.
$20,000 $5,000 = $15,000
by 50%.
B.
Direct materials costs
$10,000
A.
Identify the kaizen and materials standards in
Direct labor costs
3,000
place at the beginning of Year 2. Express the
Overhead costs
4,000
standards in both physical and financial terms.
Total
$17,000
B.
Calculate the expected cost reduction.
Less:
C.
Calculate the actual cost reduction.
Value analysis
$2,000
Process redesign
1,000
Total Costs
$14,000
ANS:
C.
Yes, Overland should manufacture the car
A.
Materials:
because the total costs of $14,000 is less than
Physical
4,000
its target cost. Its expected profit is now
standard:
bundles
$6,000 ($20,000 $14,000).
Financial
$85,000
($20 4,250)
standard:
PTS:
1
DIF:
Difficulty: Easy
Kaizen:
OBJ:
LO: 10-5
Physical
4,100
[4,250 ((4,250
NAT:
BUSPROG: Analytic
standard:
bundles
4,000) 60%)]
STA:
AICPA: FN-Decision Modeling | IMA: Strategic
Financial
$82,000
($20 4,100)
Planning | ACBSP: APC-27-Managerial Accounting
standard:
Features/Costs
KEY:
Bloom's: Application
B.
Expected cost reduction: $85,000 $82,000 =
NOT:
5 min.
$3,000
Anderson Company has the following information
C.
Actual cost reduction: $85,000 $82,500 =
concerning its direct labor:
$2,500
4,250 ((4,250 4,000) 50%) = 4,125
Direct Labor:
bundles
Standard Hours
6,500
4,125 $20 = $82,500
Actual Hours
6,350
Standard Rate
$15
Actual Rate
$18
PTS:
1
DIF:
Difficulty: Moderate
OBJ:
LO: 10-5
A.
Determine the labor rate variance and whether
NAT:
BUSPROG: Analytic
it is favorable or unfavorable.
STA:
AICPA: FN-Decision Modeling | IMA: Strategic
B.
Determine the labor efficiency variance and
Planning | ACBSP: APC-27-Managerial Accounting
whether it is favorable or unfavorable.
Features/Costs
KEY:
C.
Provide the journal entry for the labor
Bloom's: Application
variances.
NOT:
5 min.
25.

Overland Automotive Company is considering on


manufacturing a new brand of car. Given the current
product and process designs, the cost data are:
Direct materials costs (per car)
Direct labor costs (per car)
Overhead costs (per car)

$10,000
$ 3,000
$ 4,000

The company expects the selling price to be $20,000


and has set a target profit of $5,000.

ANS:
A.

LRV

= (AR SR)AH
= ($18 $15)6,350
= $19,050 U

B.

LEV

= (AH SH)SR
= (6,350 6,500)$15
= $2,250 F

C.
Work in Process
97,500
A supplier told Overland that it could purchase a couple
Labor Rate Variance
19,050
of similar components under a different brand name at
Labor Efficiency Variance
2,250
a lower price. This would result in cost savings of
Accrued Payroll
114,300
$2,000 per car. Furthermore, the company found that it
could redesign its manufacturing process to cut down
on both inspection labor and worker labor, which would PTS:
1
DIF:
Difficulty: Easy
result in cost savings of $1,000 per car.
OBJ:
LO: 10-5 | LO: 10-6

NAT:
BUSPROG: Analytic
Historical experience can provide an initial guideline for
STA:
AICPA: FN-Measurement | IMA: Performance
setting standards, but should be used with caution
Measurement | ACBSP: APC-06-Recording Transactions | because they can perpetuate existing inefficiencies.
ACBSP: APC-33-Incremental analysis
KEY:
Engineering studies can identify efficient approaches
Bloom's: Application
rigorous guidelines, but engineering standards often are
NOT:
5 min.
too rigorous. Input from operating personnel-since
operating personnel are accountable for meeting
ESSAY
standards, they should have significant input in setting
standards.
You decide
PTS:
1
DIF:
Difficulty: Moderate
1. Explain the three potential sources of quantitative
OBJ:
LO: 10-1
standards.
NAT:
BUSPROG: Communication
STA:
AICPA: FN-Decision Modeling | IMA: Strategic
ANS:
Planning | ACBSP: APC-25-Managerial
Characteristics/Terminology
KEY:
Bloom's: Comprehension
NOT:
5 min.

Das könnte Ihnen auch gefallen